You are on page 1of 99

12th Edition Assessment – Hepatic Disorder

1. The majority of blood supply to the liver, which is rich in nutrients from the gastrointestinal tract,
comes from the:
a. hepatic artery.
b. hepatic vein.
c. portal artery.
d. portal vein.

2. The liver plays a major role in glucose metabolism by:


a. producing ketone bodies.
b. synthesizing albumin.
c. participating in gluconeogenesis.
d. doing all of the above.

3. The liver synthesizes prothrombin only if there is enough:


a. vitamin A.
b. vitamin B12.
c. vitamin D.
d. vitamin K.

4. The substance necessary for the manufacture of bile salts by hepatocytes is:
a. albumin.
b. bilirubin.
c. cholesterol.
d. vitamin D.

5. The main function of bile salts is:


a. albumin synthesis.
b. fat emulsification in the intestines.
c. lipid manufacture for the transport of proteins.
d. urea synthesis from ammonia.

6. Hepatocellular dysfunction results in all of the following except:


a. decreased serum albumin.
b. elevated serum bilirubin.
c. increased blood ammonia levels.
d. increased levels of urea.

7. Jaundice becomes evident when serum bilirubin levels exceed:


a. 0.5 mg/dL.
b. 1.0 mg/dL.
c. 1.5 mg/dL.
d. 2.5 mg/dL.

8. The liver converts ammonia to urea. What level of ammonia would suggest liver failure?
a. 40 g/dL
b. 100 mg/dL
c. 200 g/dL
d. 300 mg/dL

9. The most common cause of parenchymal cell damage and hepatocellular dysfunction is:
a. infectious agents.
b. malnutrition.
c. metabolic disorders.
d. toxins.

10. Negative sodium balance is important for a patient with ascites. An example of food permitted on a
low-sodium diet is:
a. one-forth cup of peanut butter.
b. one cup of powdered milk.
c. one frankfurter.
d. two slices of cold cuts.

11. The nurse expects that the diuretic of choice for a patient with ascites would be:
a. Aldactone.
b. ammonium chloride.
c. Diamox.
d. Lasix.

12. An indicator of probable esophageal varices is:


a. hematemesis.
b. a positive guaiac test.
c. melena.
d. all of the above.

13. The mortality rate from the first bleeding episode for esophageal varices is about:
a. 10% to 15%.
b. 15% to 25%.
c. 30% to 50%.
d. 80% or higher.

14. Bleeding esophageal varices result in a decrease in:


a. nitrogen load from bleeding.
b. renal perfusion.
c. serum ammonia.
d. all of the above.

15. The initial model of therapy to treat variceal hemorrhage that decreases portal pressure and
produces constriction is:
a. Corgard.
b. Isordil.
c. Pitressin.
d. Somatostatin.

16. A person who consumes contaminated shellfish would probably develop:


a. hepatitis B.
b. hepatitis C.
c. hepatitis D.
d. hepatitis E.

17. The hepatitis virus that is transmitted via the fecal-oral route is:
a. hepatitis A virus.
b. hepatitis B virus.
c. hepatitis C virus.
d. hepatitis D virus.

18. Immune serum globulin provides passive immunity against type A hepatitis in those not vaccinated if
it is administered within 2 weeks of exposure. Immunity is effective for about:
a. 1 month.
b. 2 months.
c. 3 months.
d. 4 months.

19. Choose the correct statement about hepatitis B vaccine.


a. All persons at risk should receive active immunization.
b. Evidence suggests that the human immunodeficiency virus (HIV) may be harbored in the vaccine.
c. Booster doses are recommended every 5 years.
d. One dose in the dorsogluteal muscle is recommended.

20. Indications for postexposure vaccination with hepatitis B immune globulin include:
a. accidental exposure to HbAg-positive blood.
b. perinatal exposure.
c. sexual contact with those who are positive for HbAg.
d. all of the above exposures.

21. This hepatitis virus caused by contaminated needles shared by drug users is expected to increase
fourfold by 2015. This type of hepatitis, which is also the most common cause for liver transplantation,
is:
a. hepatitis A.
b. hepatitis B.
c. hepatitis C.
d. hepatitis D.

22. The chemical most commonly implicated in toxic hepatitis is:


a. chloroform.
b. gold compounds.
c. phosphorus.
d. all of the above hepatotoxins.

23. Acetaminophen, found in over-the-counter (OTC) drugs, is the leading cause of acute liver failure.
A popular drug containing acetaminophen is:
a. Advil.
b. Aleve.
c. Motrin.
d. Tylenol.
24. Fulminant hepatic failure may progress to hepatic encephalopathy about how many weeks after
disease onset?
a. 2 weeks
b. 4 weeks
c. 6 weeks
d. 8 weeks

25. The major causative factor in the etiology of cirrhosis is:


a. acute viral hepatitis.
b. chronic alcoholism.
c. chronic biliary obstruction.
d. infection (cholangitis).

26. Late symptoms of hepatic cirrhosis include all of the following except:
a. edema.
b. hypoalbuminemia.
c. hypokalemia.
d. hyponatremia.

27. Cirrhosis results in shunting of portal system blood into collateral blood vessels in the
gastrointestinal tract. The most common site is:
a. the esophagus.
b. the lower rectum.
c. the stomach.
d. a combination of all of the above.

28. Signs of advanced liver disease include:


a. ascites.
b. jaundice.
c. portal hypertension.
d. all of the above.

29. The most common single cause of death in patients with cirrhosis is:
a. congestive heart failure.
b. hepatic encephalopathy.
c. hypovolemic shock.
d. ruptured esophageal varices.

30. Hepatic lobectomy for cancer can be successful when the primary site is localized. Because of the
regenerative capacity of the liver, a surgeon can remove up to what percentage of liver tissue?
a. 25%
b. 50%
c. 75%
d. 90%
Assessment – Biliary Disorder

1. Bile is stored in the:


a. cystic duct.
b. duodenum.
c. gallbladder.
d. common bile duct.

2. A patient is diagnosed with gallstones in the bile ducts. The nurse knows to review the results of blood
work for a:
a. serum ammonia concentration of 90 mg/dL.
b. serum albumin concentration of 4.0 g/dL.
c. serum bilirubin level greater than 1.0 mg/dL.
d. serum globulin concentration of 2.0 g/dL.

3. The major stimulus for increased bicarbonate secretion from the pancreas is:
a. amylase.
b. lipase.
c. secretin.
d. trypsin.

4. An action not associated with insulin is the:


a. conversion of glycogen to glucose in the liver.
b. lowering of blood glucose.
c. promotion of fat storage.
d. synthesis of proteins.

5. The nurse knows that a patient with low blood sugar would have a blood glucose level of:
a. 55 to 75 mg/dL.
b. 80 to 120 mg/dL.
c. 130 to 150 mg/dL.
d. 160 to 180 mg/dL.

6. A patient with calculi in the gallbladder is said to have:


a. cholecystitis.
b. cholelithiasis.
c. choledocholithiasis.
d. choledochotomy.

7. Statistics show that there is a greater incidence of gallbladder disease for women who are:
a. multiparous.
b. obese.
c. older than 40 years of age.
d. characterized by all of the above.

8. The obstruction of bile flow due to cholelithiasis can interfere with the absorption of:
a. vitamin A.
b. vitamin B6.
c. vitamin B12.
d. vitamin C.

9. Clinical manifestations of common bile duct obstruction include all of the following except:
a. amber-colored urine.
b. clay-colored feces.
c. pruritus.
d. jaundice.

10. The diagnostic procedure of choice for cholelithiasis is:


a. x-ray.
b. oral cholecystography.
c. cholecystography.
d. ultrasonography.

11. Pharmacologic therapy is frequently used to dissolve small gallstones. It takes about how many
months of medication with UDCA or CDCA for stones to dissolve?
a. 1 to 2 months
b. 3 to 5 months
c. 6 to 8 months
d. 6 to 12 months

12. Chronic pancreatitis, commonly described as autodigestion of the pancreas, is often not detected
until what percentage of the exocrine and endocrine tissue is destroyed?
a. 10% to 25%
b. 30% to 50%
c. 60% to 75%
d. 80% to 90%

13. Mild acute pancreatitis is characterized by:


a. edema and inflammation.
b. pleural effusion.
c. sepsis.
d. disseminated intravascular coagulopathy.

14. A major symptom of pancreatitis that brings the patient to medical care is:
a. severe abdominal pain.
b. fever.
c. jaundice.
d. mental agitation.

15. The nurse should assess for an important early indicator of acute pancreatitis, which is a prolonged
and elevated level of:
a. serum calcium.
b. serum lipase.
c. serum bilirubin.
d. serum amylase.
16. Nursing measures for pain relief for acute pancreatitis include:
a. encouraging bed rest to decrease the metabolic rate.
b. teaching the patient about the correlation between alcohol intake and pain.
c. withholding oral feedings to limit the release of secretin.
d. all of the above.

17. The risk for pancreatic cancer is directly proportional to:


a. age.
b. dietary intake of fat.
c. cigarette smoking.
d. presence of diabetes mellitus.

18. With pancreatic carcinoma, insulin deficiency is suspected when the patient evidences:
a. an abnormal glucose tolerance.
b. glucosuria.
c. hyperglycemia.
d. all of the above.

19. Clinical manifestations associated with a tumor of the head of the pancreas include:
a. clay-colored stools.
b. dark urine.
c. jaundice.
d. all of the above.
Assessment – Diabetes Mellitus
1. Glucose intolerance increases with age. The incidence in those older than 65 years is:
a. 20%
b. 50%
c. 65%
d. 80%

2. The ethnic group with the lowest incidence of diabetes mellitus in the United States is:
a. African Americans.
b. Caucasians.
c. Hispanics.
d. Native Americans.

3. As a cause of death by disease in the United States, diabetes mellitus ranks:


a. first.
b. second.
c. third.
d. fourth.

4. A patient is diagnosed with type 1 diabetes. The nurse knows that all of the following are probable
clinical characteristics except:
a. ketosis-prone.
b. little endogenous insulin.
c. obesity at diagnoses.
d. younger than 30 years.

5. A patient who is diagnosed with type 1 diabetes mellitus would be expected to:
a. be restricted to an American Diabetic Association diet.
b. have no damage to the islet cells of the pancreas.
c. need exogenous insulin.
d. need to receive daily doses of a hypoglycemic agent.

6. Possible risk factors associated with type 1 diabetes mellitus include:


a. an autoimmune susceptibility to diabetogenic viruses.
b. environmental factors.
c. the presence of human leukocyte antigen (HLA).
d. all of the above.

7. Clinical manifestations associated with a diagnosis of type 1 diabetes mellitus include all of the
following except:
a. hypoglycemia.
b. hyponatremia.
c. ketonuria.
d. polyphagia.

8. The nurse is asked to assess a patient for glucosuria. The nurse would secure a specimen of:
a. blood.
b. sputum.
c. stool.
d. urine.

9. Knowing that gluconeogenesis helps to maintain blood levels, a nurse should:


a. document weight changes because of fatty acid mobilization.
b. evaluate the patient’s sensitivity to low room temperatures because of decreased adipose tissue
insulation.
c. protect the patient from sources of infection because of decreased cellular protein deposits.
d. do all of the above.

10. A nurse is assigned to care for a patient who is suspected of having type 2 diabetes mellitus. Clinical
manifestations for which the nurse should assess include:
a. blurred or deteriorating vision.
b. fatigue and muscle cramping.
c. wounds that heal slowly or respond poorly to treatment.
d. all of the above.

11. There seems to be a strong positive correlation between type 2 diabetes mellitus and:
a. hypotension.
b. kidney dysfunction.
c. obesity.
d. sex.

12. The lowest fasting plasma glucose level suggestive of a diagnosis of diabetes is:
a. 90 mg/dL.
b. 115 mg/dL.
c. 126 mg/dL.
d. 180 mg/dL.

13. The most sensitive test for diabetes mellitus is the:


a. fasting plasma glucose.
b. 2-hour postload glucose.
c. intravenous glucose.
d. urine glucose.

14. A female diabetic patient who weighs 130 lb has an ideal body weight of 116 lb. For weight
reduction of 2 lb/week, her daily caloric intake should be approximately:
a. 1,000 calories.
b. 1,200 calories.
c. 1,500 calories.
d. 1,800 calories.

15. The nurse should encourage exercise in the management of diabetes, because it:
a. decreases total triglyceride levels.
b. improves insulin utilization.
c. lowers blood glucose.
d. accomplishes all of the above.
16. Self-monitoring of blood glucose is recommended for patients with:
a. abnormal renal glucose thresholds.
b. hypoglycemia without warning symptoms.
c. unstable diabetes.
d. all of the above conditions.

17. An example of a commonly administered intermediate-acting insulin is:


a. NHP.
b. Iletin II.
c. Humalog.
d. Humulin U.

18. The nurse knows that an intermediate-acting insulin should reach its “peak” in:
a. 1 to 2 hours.
b. 3 to 4 hours.
c. 4 to 12 hours.
d. 16 to 20 hours.

19. Insulin pumps in use today:


a. can deliver a premeal dose (bolus) of insulin before each meal.
b. deliver a continuous basal rate of insulin at 0.5 to 2.0 units/h.
c. prevent unexpected savings in blood glucose measurements.
d. are capable of doing all of the above.

20. A probable candidate for diabetic management with oral antidiabetic agents is the patient who is:
a. non–insulin-dependent.
b. stable and not prone to ketosis.
c. unable to be managed by diet alone.
d. characterized by all of the above.

21. An example of a first-generation sulfonylurea used in type 2 diabetes is:


a. Dia-Beta
b. Glyset
c. Starlix
d. Tolinase

22. The nurse should expect that insulin therapy will be temporarily substituted for oral antidiabetic
therapy if the diabetic patient:
a. develops an infection with fever.
b. suffers trauma.
c. undergoes major surgery.
d. develops any or all of the above.

23. The tissue area that provides the fastest absorption rate for regular insulin is believed to be the:
a. abdominal area.
b. anterior thigh.
c. deltoid area.
d. gluteal site.
24. Rotation sites for insulin injection should be separated from one another by 2.5 cm (1 in) and should
be used only once every:
a. third day.
b. week.
c. 2 to 3 weeks.
d. 2 to 4 weeks.

25. Hypoglycemia, an abnormally low blood glucose concentration, occurs with a glucose level that is:
a. lower than 50 to 60 mg/dL.
b. between 60 and 80 mg/dL.
c. between 75 and 90 mg/dL.
d. 95 mg/dL.

26. A clinical feature that distinguishes a hypoglycemic reaction from a ketoacidosis reaction is:
a. blurred vision.
b. diaphoresis.
c. nausea.
d. weakness.

27. The nurse knows that treatment modalities for diabetic ketoacidosis should focus on management
of:
a. acidosis.
b. dehydration.
c. hyperglycemia.
d. all of the above.

28. The major electrolyte of concern in the treatment of diabetic ketoacidosis is:
a. calcium.
b. magnesium.
c. potassium.
d. sodium.

29. Mortality rates for patients with diabetes are positively correlated with atherosclerotic
complications, espe
cially in the coronary arteries, which account for about what percentage of all deaths in these patients?
a. 10%
b. 30%
c. 40%
d. 60%

30. Macrovascular disease has a direct link with:


a. hypertension.
b. increased triglyceride levels.
c. obesity.
d. all of the above.
31. Clinical nursing assessment for a patient with microangiopathy who has manifested impaired
peripheral arterial circulation includes all of the following except:
a. integumentary inspection for the presence of brown spots on the lower extremities.
b. observation for paleness of the lower extremities.
c. observation for blanching of the feet after the legs are elevated for 60 seconds.
d. palpation for increased pulse volume in the arteries of the lower extremities.

32. With nonproliferative (background) retinopathy, examination of the retina may reveal:
a. leakage of fluid or serum (exudates).
b. microaneurysms.
c. focal capillary single closure.
d. all of the above pathologic changes.

33. A diagnostic manifestation of proliferative retinopathy is:


a. decreased capillary permeability.
b. microaneurysm formation.
c. neovascularization into the vitreous humor.
d. the leakage of capillary wall fragments into surrounding areas.

34. A nurse caring for a diabetic patient with a diagnosis of nephropathy would expect the urinalysis
report to indicate:
a. albumin.
b. bacteria.
c. red blood cells.
d. white blood cells.

35. With peripheral neuropathy, a diabetic patient has limited sensitivity to:
a. heat.
b. pain.
c. pressure.
d. all of the above.

36. Nursing care for a diabetic patient with peripheral neuropathy includes:
a. assessing pain patterns to rule out peripheral
vascular insufficiency.
b. inspecting the feet for breaks in skin integrity.
c. palpating the lower extremities for temperature variations.
d. all of the above.

37. During surgery, glucose levels will rise, because there is an increased secretion of:
a. cortisol.
b. epinephrine.
c. glucagon.
d. all of the above.
38. The nurse expects that a type 1 diabetic patient may receive what percentage of his or her usual
morning dose of insulin preoperatively?
a. 10% to 20%
b. 25% to 40%
c. 50% to 60%
d. 85% to 90%
Iggy ch 58, 59 Quiz
Study online at https://quizlet.com/_9gkz85

A patient with decompensated cirrhosis


is hich at risk for which complications?
SATA. a. Jaundice
a. Jaundice b. Esophageal varices
b. Esophageal varices c. Coagulation defects
c. Coagulation defects e. Spontaneous bacterial peritonitis
d. Hepatitis A virus (HAV) f. Ascites
e. Spontaneous bacterial peritonitis
f. Ascites
What is the most common cause for
Laennec's cirrhosis?
a. Hepatitis C virus (HPC)
d. Chronic alcoholism
b. Chronic biliary obstruction
c. Autoimmune disorders
d. Chronic alcoholism
The nurse is assessing a patient with
massive ascites. What related complica-
tion must the nurse monitor for with this
patient?
a. Bleeding due to fragile, thin-walled
c. Increased ascites due to sodium and
veins
water retention
b. Hematemesis due to absence of clot-
ting factors
c. Increased ascites due to sodium and
water retention
d. Bruising due to low platelet count
When admitting the patient with cirrho-
sis, the nurse assesses for which condi-
tion related to splenomegaly as possible
complications of the disease?
a. Thrombocytopenia
a. Thrombocytopenia
b. Bleeding esophageal varices
c. Hepatorenal syndrome
d. Portal hypertensive gastropathy
Patients with cirrhosis are susceptible
to bleeding and easy bruising because
there is a decrease in the production of

1 / 31
Iggy ch 58, 59 Quiz
Study online at https://quizlet.com/_9gkz85
bile in the liver, preventing the absorption
of which vitamin?
a. Vitamin A
d. Vitamin K
b. Vitamin D
c. Vitamin E
d. Vitamin K
Which key points does the nurse include
when teachnig the patient with cirrhosis
and his family about drug therapy before
discharge? SATA. a. "Do not take over the counter med-
a. "Do not take over the counter med- ications unless approved by your health
ications unless approved by your health care provider."
care provider." c. "The lactulose syrup should cause you
b. "The beta blocker called propranolol to have two to three bowel movements
will cause your heart rate to increase." every day."
c. "The lactulose syrup should cause you d. "Take your furosemide early in the dau
to have two to three bowel movements so that it does not keep you up at night."
every day." e. "Report any muscle weakness or light-
d. "Take your furosemide early in the dau headedness to your health care provider
so that it does not keep you up at night." right away."
e. "Report any muscle weakness or light- f. "Your health care provider may pre-
headedness to your health care provider scribe a potassium supplement to re-
right away." place losses."
f. "Your health care provider may pre-
scribe a potassium supplement to re-
place losses."
The nurse identifies which laboratory
value as the usual indication of hepativ
encephalopathy?
a. Elevated sodium level b. Elevated ammonia level
b. Elevated ammonia level
c. Increased blood urea nitrogen (BUN)
d. Increased clotting time
The nurse is assessing a male patient
with cirrhosis. Which male-specific char-
acteristics does the nurse expect to find?
SATA.
a. Gynecomastia
2 / 31
Iggy ch 58, 59 Quiz
Study online at https://quizlet.com/_9gkz85
b. Testicular atrophy
c. Ascites a. Gynecomastia
d. Impotence b. Testicular atrophy
e. Spider angiomas d. Impotence
f. Petechiae
Which assessment finding indicates
neurologic function deterioration in a pa-
tient with stage II cirrhosis?
a. Fector hepaticus b. Asterixis
b. Asterixis
c. Palmar erythema
d. Icterus
Which intervention should the nurse del-
egate to the unlicensed assistive per-
sonnel (UAP) when caring for a patient
with cirrhosis experiencing pruritis?
a. Apply lotion to soothe the patient's
skin a. Apply lotion to soothe the patient's
b. Use lots of soap and hot water to skin
cleanse the skin
c. Assess the patient for signs of skin
infection
d. Encourage the patient to use distrac-
tion to avoid scratching
Which elevated laboratory test results
indicate hepatic cell destruction? SATA.
a. Elevated serum aspartate amino- a. Elevated serum aspartate aminotrans-
transferase (AST) ferase (AST)
b. Elevated serum alanine aminotrans- b. Elevated serum alanine aminotrans-
ferase (ALT) ferase (ALT)
c. Elevated lactate dehydrogenase c. Elevated lactate dehydrogenase
(LDH) (LDH)
d. Decreased serum total bilirubin f. Increased International Normalized
e. Increased fecal urobilinogen Ratio (INR)
f. Increased International Normalized
Ratio (INR)
A patient is schedule for a procedure
to place a stent in the biliary tract. For
3 / 31
Iggy ch 58, 59 Quiz
Study online at https://quizlet.com/_9gkz85
which procedure does the nurse provide
patient teaching?
a. Esophagogastroduodenoscopy
(EGD) b. Endoscopic retrograde choliangiopan-
b. Endoscopic retrograde choliangiopan- creatography (ERCP)
creatography (ERCP)
c. Upper gastrointestinal (GI) series
d. Cholangiogram
The nurse is teaching a patient with
cirrhosis about nutrition therapy. Which
statement by the patient indicates teach-
ing has been effective?
a. "I will only use table salt with my dinner b. "I will read the sodium content labels
meal." on all food and beverages."
b. "I will read the sodium content labels
on all food and beverages."
c. "I will avoid the use of vinegar."
d. "I will not take vitamin supplements."
When preparing a patient for paracente-
sis, what does the nurse do? SATA.
a. Ask the patient to void before the pro-
a. Ask the patient to void before the pro-
cedure
cedure
c. Weigh the patient before the proce-
b. Place the patient in the supine position
dure
c. Weigh the patient before the proce-
d. Obtain the patient's heart rate
dure
e. Assess the patient's respiratory rate
d. Obtain the patient's heart rate
f. Obtain the patient's blood pressure
e. Assess the patient's respiratory rate
f. Obtain the patient's blood pressure
A patient will undergo an abdominal
paracentesis. Which factor prodives an
additional safety measure?
a. The procedure is performed using ul-
trasound a. The procedure is performed using ul-
b. The procedure is performed at the trasound
bedside
c. A trocar is inserted into the peritoneal
cavity
d. General anesthesia is administered
4 / 31
Iggy ch 58, 59 Quiz
Study online at https://quizlet.com/_9gkz85
The student nurse is caring for a patient
with cirrhosis. Which action by the stu-
dent nurse causes the supervising nurse
to intervene?
a. Uses a straight-edge razor to shave a. Uses a straight-edge razor to shave
the patient the patient
b. Monitors for orthostatic changes of
blood pressure
c. Acoids intramuscular injections
d. Uses a toothette for oral care
The nurse who is assessing a pa-
tient with portal-systemic encephalopa-
thy finds that the patient has fetor he-
paticus, a positive Babinski's sign, and
seizures, but no asterixis. The nurse
identifies the patient as being in which d. Stage IV comatose
stage of portal-systemic failure?
a. Stage I prodromal
b. Stage II impending
c. Stage III stuporus
d. Stage IV comatose
Which statements about a patient wih
cirrhosis and esophageal varices are ac-
curate? SATA.
a. All patients with cirrhosis should be
a. All patients with cirrhosis should be
screened for esophageal varices to de-
screened for esophageal varices to de-
tect them before they bleed
tect them before they bleed
b. Bleeding esophageal varices are a
b. Bleeding esophageal varices are a
medical emergency
medical emergency
c. Esophageal baloon tamponade is of-
d. A nonselective beta blocker such
ten used to control bleeding esophageal
as propranolol is prescribed to prevent
varices
varices from bleeding
d. A nonselective beta blocker such
e. Bleeding esophageal varices can be
as propranolol is prescribed to prevent
managed by endoscopic variceal ligation
varices from bleeding
e. Bleeding esophageal varices can be
managed by endoscopic variceal liga-
tion

5 / 31
Iggy ch 58, 59 Quiz
Study online at https://quizlet.com/_9gkz85
f. The bleeding appears as dark coffee
grounds in emesis or stool
The nurse is teaching a patient with
cirrhosis about lactulose therapy. Which
statement by the patient indicates the
teaching had been effective?
a. "This therapy will promote the removal
a. "This therapy will promote the removal
of ammonia in my stool."
of ammonia in my stool."
b. "Constipation is a frequent side effect
of this therapy."
c. "I will know the therapy is working
when I am less itchy."
d. "The drug tastes bitter and is watery."
How is neomycin sulfate used to treat
patients with cirrhosis?
a. It treats the current infection the pa-
tient has d. It decreases the rate of ammonia pro-
b. It prevents future infections of the liver duction
c. It restores normal function to the liver
d. It decreases the rate of ammonia pro-
duction
The patient with liver cancer will be dis-
charged with a tunneled ascites drain.
What statements by the patient indicate
an understanding of the purpose of this
device? SATA.
b. "I will not remove more than 2000 mL
a "I will have this drain until I am able to
of fluid at a time."
get the tumor removed."
c. "The drain will make breathing more
b. "I will not remove more than 2000 mL
comfortable for me after some fluid is
of fluid at a time."
removed."
c. "The drain will make breathing more
e. "This drain will be useful to remove
comfortable for me after some fluid is
fluid from my belly when there is too
removed."
much."
d. "After I drain off the extra fluid, I can
remove the drain."
e. "This drain will be useful to remove
fluid from my belly when there is too
much."
6 / 31
Iggy ch 58, 59 Quiz
Study online at https://quizlet.com/_9gkz85
f. "I will flush the tunneled ascites drain
twice a day with normal saline."
Which statements about hepatitis are
accurate? SATA.
a. Hepatitis D is the leading cause of
cirrhosis and liver failure in the US
b. Hepatitis A is spread through the fe- b. Hepatitis A is spread through the fe-
cal-oral route cal-oral route
c. Hepatitis B can be transmitted through c. Hepatitis B can be transmitted through
unprotected sexual intercourse unprotected sexual intercourse
d. Hepatitis carries have chronic obvious f. Hepatitis D only occurs with hepatitis B
signs of hepatitis B to cause viral replication
e. Hepatitis C is transmitted by casual
contact or intimate household contact
f. Hepatitis D only occurs with hepatitis B
to cause viral replication
When teaching a group of adult patients
about measures for preventing hepati-
tis A (HAV), which information does the
nurse include? SATA.
a. Perform proper handwashing, espe-
a. Perform proper handwashing, espe-
cially after handling shellfish
cially after handling shellfish
b. Receive immune globulin within 14
b. Receive immune globulin within 14
days if exposed to the virus
days if exposed to the virus
c. Receive the HAV vaccine before trav-
c. Receive the HAV vaccine before trav-
eling to Mexico or the Carribean
eling to Mexico or the Carribean
d. After exposure, HAV symptoms al-
e. Receive the vaccine if working in a
ways let the patient know something is
long-term care facility
wrong
e. Receive the vaccine if working in a
long-term care facility
f. Avoid unprotected sex with a person
who has HAV
Which people need immunization
against hepatitis B (HBV)? SATA. a. People who have unprotected sex with
a. People who have unprotected sex with more than one partner
more than one partner b. Men who have sex with men
b. Men who have sex with men
7 / 31
Iggy ch 58, 59 Quiz
Study online at https://quizlet.com/_9gkz85
c. Any patient scheduled for a surgical
procedure d. Firefighters
d. Firefighters e. Health care providers
e. Health care providers f. Patients prescribed immunosuppres-
f. Patients prescribed immunosuppres- sant drugs
sant drugs
What is the major source of hepatitis B
transmission to health care workers?
a. Improper handwashing
b. Needle sticks
b. Needle sticks
c. Touching contaminated surfaces
d. Contact with infected stool
How many vaccine injections does a
health care worker usually need too be
protected with the hepatitis B vaccine?
a. 1 c. 3
b. 2
c. 3
d. 4
Which actions will help prevent viral he-
patitis in health care workers? SATA.
a. Wash hands before and after each
a. Wash hands before and after each
patient
patient
b. Use needleless systems
b. Use needleless systems
c. Use contact and respiratory precau-
d. After exposure to hepatitis A, get im-
tions
munoglobulin
d. After exposure to hepatitis A, get im-
e. Report all cases of hepatitis to the
munoglobulin
health department
e. Report all cases of hepatitis to the
health department
f. Wear gloves during all patient contacts
Which laboratory test results indicates
permanent immunity to hepatitis A?
a. Immunoglobulin G (IgG) antibodies
b. Immunoglobulin M (IgM) antibodies a. Immunoglobulin G (IgG) antibodies
c. A positive enzyme-linked immunosor-
bent assay (ELISA)
d. The presence of anti-HAV antibodies
8 / 31
Iggy ch 58, 59 Quiz
Study online at https://quizlet.com/_9gkz85

Which antiviral drugs are given to pa-


tients with chronic hepatitis B virus?
SATA.
a. Lamivudine
a. Lamivudine
b. Entecavir
b. Entecavir
c. Tenofovir
c. Tenofovir
e. Adefovir
d. Oral ribavirin
e. Adefovir
f. Telaprevir
Which conditions place a patient at high
risk for the development of fatty liver
(steatosis)? SATA.
b. Diabetes mellitus
a. Hypertension
c. Obesity
b. Diabetes mellitus
d. Elevated lipid profile
c. Obesity
e. Alcohol abuse
d. Elevated lipid profile
e. Alcohol abuse
f. Hepatitis A
In performing an assessment on a pa-
tient with liver trauma, what does the
nurse expect to find? SATA.
a. Right upper quadrant pain
a. Right upper quadrant pain
c. Guarding of the abdomen
b. Increased blood pressure
e. Kehr's sign
c. Guarding of the abdomen
f. Abdominal rigidity
d. Bradypnea
e. Kehr's sign
f. Abdominal rigidity
The nurse is assessing a patient with
liver trauma and finds that the patient is
confused, with a blood pressure of 86/50
mmHg, heart rate of 128 per minute, and
cool, clammy skin. What does the nurse
b. Liver hemorrhage
suspect?
a. Septic shock
b. Liver hemorrhage
c. Liver cancer
d. GI bleeding

9 / 31
Iggy ch 58, 59 Quiz
Study online at https://quizlet.com/_9gkz85
What test result is the tumor marker for
cancers of the liver?
a. Decreased alkaline phosphatase
d. Increased alpha-fetoprotein (AFP)
b. Increased serum ammonia
c. Decreased serum total bilirubin
d. Increased alpha-fetoprotein (AFP)
Which treatment offers the patient with
liver cancer the possibility of long-term
survival?
a. Chemotherapy c. Liver transplantation
b. Selective internal radiation therapy
c. Liver transplantation
d. Hepatic arterial embolization
What is the priority fovus in caring for a
patient with advanced liver cancer?
a. Hospice and end-of-life care
b. Getting placed on the liver transplant
list a. Hospice and end-of-life care
c. Hepatic arterial infusion of chemother-
apy
d. Cryotherapy to freeze and destroy liv-
er tumors
Administration of which drug has greatly
improved the success of organ trans-
plants?
a. Telaprevir d. Cyclosporine
b. Entecavir
c. Tenofovir
d. Cyclosporine
The patient who had a liver transplant
develops a heart rate of 134/minute,
temperature of 102F (38.8C), jaundiced
skin, and right upper quadrant pain.
c. Liver transplant rejection
What does the nurse suspect?
a. Liver infection
b. Hypovolemic shock
c. Liver transplant rejection

10 / 31
Iggy ch 58, 59 Quiz
Study online at https://quizlet.com/_9gkz85
d. Liver trauma from the transplant
surgery
Which procedure uses energy waves to
heat cancer cells and kill them?
a. Cryotherapy
b. Selective internal radiation therapy d. Radiofrequency ablation (RFA)
(SIRT)
c. Hepatic artery embolization
d. Radiofrequency ablation (RFA)
Which patients would not be considered
candidates for a liver transplant? SATA.
a. Patient with metastatic tumors
a. Patient with metastatic tumors
b. Patient with type 2 diabetes
c. Patient with severe respiratory disease
c. Patient with severe respiratory dis-
e. Patient with advanced cardiac disease
ease
f. Patient who is unable to follow instruc-
d. Patient with chronic liver disease
tions
e. Patient with advanced cardiac disease
f. Patient who is unable to follow instruc-
tions
The nurse is teaching a patient with cir-
rhosis about nutrition therapy. Which key
points must the nurse include? SATA.
a. Do not use table sale a. Do not use table sale
b. Adding salt when cooking is accept- c. Eat small frequent meals
able d. Drink supplemental liquids such as
c. Eat small frequent meals Ensure
d. Drink supplemental liquids such as e. Be sure to take a multlivitamin every
Ensure day
e. Be sure to take a multlivitamin every
day
f. Avoid foods that are rich in vitamin K
The patient with cirrhosis is prescribed
furosemide 60 mg orally each morning.
Which patient care concept is at risk for
d. Fluid and electrolyte balance
this patient?
a. Comfort
b. Cellular regulation

11 / 31
Iggy ch 58, 59 Quiz
Study online at https://quizlet.com/_9gkz85
c. Immunity
d. Fluid and electrolyte balance
Which factors may lead to hepatic en-
cephalopathy in patients with cirrhosis?
SATA.
a. High-protein diet
a. High-protein diet
c. Infection
b. Hypervolemia
d. Constipation
c. Infection
f. Use of illicit drugs
d. Constipation
e. Hyperkalemia
f. Use of illicit drugs
The nurse is teaching a young woman
about cirrhosis prevention by limiting al-
cohol intake. What is the nurse's best
advice?
a. "As few as two or three drinks per day
over 10 years can lead to cirrhosis."
a. "As few as two or three drinks per day
b. "You should be all right as long as you
over 10 years can lead to cirrhosis."
drink less than five drinks per day."
c. "Binge drinking, rather than drinking
every day, reduces your risk for hepatitis
or fatty liver."
d. "The amoung of alcohol that causes
cirrhosis does not vary by gender."
The nurse is providing care for a pa-
tient with cirrhosis who has massive
ascites and has developed hepatopul-
monary syndrome. Which elements of
nursing care are appropriate for this pa- a. Auscultate lungs every 4-8 hours for
tient? SATA. crackles
a. Auscultate lungs every 4-8 hours for b. Monitor the patient's oxygen saturation
crackles d. Apply oxygen therapy
b. Monitor the patient's oxygen satura- e. Weigh the patient every day
tion
c. Elevate the head of the bed 15 de-
grees
d. Apply oxygen therapy

12 / 31
Iggy ch 58, 59 Quiz
Study online at https://quizlet.com/_9gkz85
e. Weigh the patient every day
f. Lower the patient's legs and feet
The patient who needs a liver transplant
asks the nurse where the livers come
from. What is the nurse's best response?
a. "Most commonly they come from fam-
ily members."
c. "Trauma victims are where most donor
b. "Often they are harvested from cadav-
livers come from."
ers."
c. "Trauma victims are where most donor
livers come from."
d. "It is best if the liver comes from a
blood relative."
The patient who just had a liver trans-
plant develops oozing around two IV
sites as well as some new bruising. What
is the nurse's best first action?
a. Apply pressure to the IV sites d. Notify the surgeon immediately
b. Measure the size of the bruises
c. Document these findings as the only
action
d. Notify the surgeon immediately
The nurse is caring for a patient with
acute viral hapatitis. What is the major
care concern at this time?
a. Providing three small meals a day
b. Alternating periods of activity with pe- b. Alternating periods of activity with pe-
riods of rest riods of rest
c. Monitoring for the development of
jaundiced skin
d. Teaching the patient the importance of
avoiding alcohol intake
A patient is admitted with obstructive
jaundice. Which sign/symptom does the
nurse expect to find upon assessment of
a. Pruritis
the patient?
a. Pruritis
b. Pale urine in increased amounts
13 / 31
Iggy ch 58, 59 Quiz
Study online at https://quizlet.com/_9gkz85
c. Pink discoloration of sclera
d. Dark, tarry stools
The daughter of a patient with cholelithi-
asis has heard that there is a genetic
disposition for cholelithiasis. The daugh-
er asks the nurse about the risk factors.
How does the nurse respond?
a. "There is no evidence that first-degree
relatives have an increased risk for this c. "Hormone replacement therapy has
disease." been associated with increased risk for
b. "Cholelithiasis is seen more frequently cholelithiasis."
in patients who are underweight."
c. "Hormone replacement therapy has
been associated with increased risk for
cholelithiasis."
d. "Patients with diabetes mellitus are at
increased risk for cholelithiasis."
Which patient would the nurse assess as
low risk for the development of gallblad-
der disorders?
a. Patient with sickle cell anemia
c. Patient who is 20 years old and male
b. Patient who is Mexican American
c. Patient who is 20 years old and male
d. Patient with a history of prolonged
parenteral nutrition
The nurse on a medical-surgical unit is
caring for several patients with acute
cholecystitis. Which task is best to dele-
gate to the UAP?
a. Obtain the patient's vital signs
b. Determine if any foods are not tolerat- a. Obtain the patient's vital signs
ed
c. Assess what measures relieve the ab-
dominal pain
d. Ask the patients to describe their daily
activity or exercise routines
Which are common manifestations of
acute cholecystitis? SATA.
14 / 31
Iggy ch 58, 59 Quiz
Study online at https://quizlet.com/_9gkz85
a. Anorexia
b. Ascites a. Anorexia
c. Eructation c. Eructation
d. Steatorrhea e. Jaundice
e. Jaundice f. Rebound tenderness
f. Rebound tenderness
The nurse is assessing a patient with
acute cholecystitis whose abdominal
paint is severe. The patient is pale,
is diaphoretic, and describes extreme
fatigue. Vital signs are: heart rate
of 118/minute, BP 95/70, respirations
32/min, temperature 101F (38.33C).
What is the nurse's priority action at this
c. Notify the patient's health care
time?
provider
a. Instruct the UAP to reposition the pa-
tient
b. Auscultate the patient's abdomen in all
four quadrants
c. Notify the patient's health care
provider
d. Administer the ordered opioid anal-
gesic
The health care provider has assessed
a patient's abdomen and found rebound
tenderness on deep palpation. What
does the nurse recognize?
d. Blumberg's sign
a. Steatorrhea
b. Eructation
c. Biliary colic
d. Blumberg's sign
A patient is scheduled for tests to verify
the medical diagnosis of cholecystitis.
For which diagnostic test does the nurse
b. Ultrasonography of the right upper
provide patient teaching?
quadrant
a. Extracorporeal shock wave lithotripsy
(ESWL)
b. Ultrasonography of the right upper
15 / 31
Iggy ch 58, 59 Quiz
Study online at https://quizlet.com/_9gkz85
quadrant
c. Endoscopic retrograde cholangiopan-
creatography (ERCP)
d. Serum level os aspartate aminotrans-
ferase (AST)
Which type of drug is used to treat acute
severe biliary pain?
a. Acetaminophen
b. Nonsteroidal antiinflammatory drugs d. Opioids
(NSAIDs)
c. Antiemetics
d. Opioids
The nurse is administering ketorolac to
a 78-year-old patient for mild to mod-
erate biliary pain management. Which
assessment finding indicates the patient
is experiencing a side effect of this drug? a. Gastrointestinal upset
a. Gastrointestinal upset
b. Ventricular cardiac dysrhythmias
c. Decreased urinary output
d. Jaundice
The nurse is caring for an older adult pa-
tient with acute biliary pain. Which drug
order does the nurse question?
a. Ketorolac b. Meperidine
b. Meperidine
c. Morphine
d. Hydromorphone
Which factor renders a patient the least
liekly to benefit from extracorporeal
shock wave lithotripsy (ESWL) for the
treatment of gallstones?
a. Height 5 feet 10 inches, 325 lbs
a. Height 5 feet 10 inches, 325 lbs
b. Cholesterol-based stones
c. Height 5 feet 7 inches, 138 lbs
d. Small gallstones

16 / 31
Iggy ch 58, 59 Quiz
Study online at https://quizlet.com/_9gkz85
Which statements are true regarding la-
paroscopic cholecystectomy? SATA.
a. Laparoscopic cholecystectomy is con-
sidered the 'gold standard' and is per-
formed far more often than the traditional
open approach.
a. Laparoscopic cholecystectomy is con-
b. Patients with chronic lung disease or
sidered the 'gold standard' and is per-
heart failure who are unable to tolerate
formed far more often than the traditional
the oxygen used in the laparoscopic pro-
open approach.
cedure are examples of patients who
c. Removing the gallbladder with the la-
have the open surgical approach (ab-
paroscopic technique reduces the risk of
dominal laparotomy).
wound complications.
c. Removing the gallbladder with the la-
d. Patients who have their gallbladders
paroscopic technique reduces the risk of
removed by the laparoscopic technique
wound complications.
should be taught the importance of early
d. Patients who have their gallbladders
ambulation to promote absorption of car-
removed by the laparoscopic technique
bon dioxide
should be taught the importance of ear-
f. After a laparoscopic cholecystectomy,
ly ambulation to promote absorption of
assess the patient;s oxygen saturation
carbon dioxide
level frequently until the effects of the
e. Use of laparoscopic cholecystectomy
anesthesia have passed
puts the patient at increased risk for bile
duct injuries
f. After a laparoscopic cholecystectomy,
assess the patient;s oxygen saturation
level frequently until the effects of the
anesthesia have passed
Which statement about the care of a pa-
tient with a Jackson-Pratt (JP) drain after
a traditional cholecystectomy is true?
a. The patient is maintained in the prone
position
d. Serosanguineous drainage stained
b. When the patient is allowed to eat, the
with bile is expected for 24 hours
JP drain is clamped continuously
c. The JP drain is irrigated every hour for
the first 24 hours
d. Serosanguineous drainage stained
with bile is expected for 24 hours

17 / 31
Iggy ch 58, 59 Quiz
Study online at https://quizlet.com/_9gkz85
A female patient is to have her gallblad-
der removed by natural orifice translumi-
nal endoscopic surgery. What does the
nurse teach about this surgery?
a. The surgeron will use powerful shock
waves to break up the gallstones
b. The surgeon will insert a transhepat- c. The surgeon will use a vaginal ap-
ic biliary catheter to open blocked bile proach to remove the gallbladder
ducts
c. The surgeon will use a vaginal ap-
proach to remove the gallbladder
d. The surgeon will inject ursodeoxy-
cholic acide to dissolve any remaining
gallstone fragments
After removal of the gallbladder, a pa-
tient experiences abdominal pain with
vomiting for several weeks. What does
the nurse recognize?
d. Postcholecystectomy syndrome
a. Chronic cholecystitis
b. Recurrence of acute cholecystitis
c. Unremoved gallstones
d. Postcholecystectomy syndrome
The patient with acute cholecystitis has
a pacemaker. Which diagnostic test is
contraindicated?
a. Extracorporeal shock wave lithotripsy
(ERCP) b. Magnetic resonance cholangiopan-
b. Magnetic resonance cholangiopan- creatography (MRCP)
creatography (MRCP)
c. Ultrasonography of the right upper
quadrant
d. Hepatobiliary (HIDA) scan
The nurse is evaluating electrolyte val-
ues for a patient with acute pancreatitis
and notes that the serum calcium is 6.8
mEq/L. How does the nures interpret this
finding?
a. Within normal limits considering the
18 / 31
Iggy ch 58, 59 Quiz
Study online at https://quizlet.com/_9gkz85
diagnosis of acute pancreatitis
b. A result of the body not being able to
use bound calcium
b. A result of the body not being able to
c. A protective measure that will reduce
use bound calcium
the risk of complications
d. Full compensation of the parathyroid
gland
Disseminated intravascular coagulation
(DIC) is a complication of pancreatitis.
What pathophysiology leads to this com-
plication?
a. Hypovolemia
d. Consumption of clotting factors and
b. Perotoneal irritation and seepage of
formation of microthrombi
pancreatic enzymes
c. Disruption of alveolar-capillary mem-
brane
d. Consumption of clotting factors and
formation of microthrombi
The patient with acute pancreatitis ex-
periences abdominal pain. What is the
best intervention to begin managing this
pain?
a. IV opioids by means of patient-con-
trolled analgesia (PCA) a. IV opioids by means of patient-con-
b. Oral opioids such as morphine sulfate trolled analgesia (PCA)
given as needed
c. Intramuscular opioids given every 6
hours
d. Oral hydromorphone (Dilaudid) given
twice a day
The patient comes to the emergency
department (ED) with severe abdomi-
nal pain in the midepigastric area. The
patient states that the pain began sud-
c. Acute pancreatitis
denly, is continuous, and radiates to his
back, and is worse when he lies flat on
his back. What condition does the nurse
suspect?
19 / 31
Iggy ch 58, 59 Quiz
Study online at https://quizlet.com/_9gkz85
a. Acute cholecystitis
b. Pancreatic cancer
c. Acute pancreatitis
d. Pancreatic pseudocyst
Which diagnostic test is the most ac-
curate in verifying a diagnosis of acute
pancreatitis?
a. Trypsin b. Lipase
b. Lipase
c. Alkaline phosphatase
d. Alanine aminotransferase
A patent with acute pancreatitis is at risk
for the development of paralytic (ady-
namic) ileus. Which action provides the
nurse with the best indication of bowel
function?
a. Observing contents of the nasogastric
drainage c. Asking the patient if he or she has
b. Weighing the patient every day at the passed flatus or had a stool
same time
c. Asking the patient if he or she has
passed flatus or had a stool
d. Obtaining a computed tomography
(CT) scan of the abdomen with contrast
medium
Which condition is most likely to be treat-
ed with antibiotics?
a. Cancer of the gallbladder
d. Acute necrotizing pancreatitis
b. Acute cholelithiasis
c. Chronic pancreatitis
d. Acute necrotizing pancreatitis
The nurse has instructed a patient in
the recovery phase of acute pancreatitis
about diet therapy. Which statement by
the patient indicates that teaching has
been successful?
a. "I will eat the usual three meals a day
that I am used to."
20 / 31
Iggy ch 58, 59 Quiz
Study online at https://quizlet.com/_9gkz85
b. "I am eating tacos for my first meal
back home."
c. "I will avoid eating chocolate and drink- c. "I will avoid eating chocolate and drink-
ing coffee." ing coffee."
d. "I will limit the amount of protein in my
diet."
The nursing student is caring for a pa-
tient with chronic pancreatitis who is re-
ceiving pancreatic enzyme replacement
therapy. Which statement by the student
indicates the need for further study con-
cerning this therapy?
a. "The enzymes will be administered
c. "If the patient has difficulty swallowing
with meals."
the enzyme preparation, I will crush it
b. "The patient will take the drugs with a
and mix it with foods."
glass of water."
c. "If the patient has difficulty swallowing
the enzyme preparation, I will crush it
and mix it with foods."
d. "The effectiveness of pancreatic en-
zyme treatment is monitored by the fre-
quency and fat content of stools."
Which statements about pancreatic can-
cer are accurate? SATA.
a. Venous thromboembolism (VTE) is a
common comlication of pancreatic can- a. Venous thromboembolism (VTE) is a
cer common comlication of pancreatic can-
b. Pancreatic cancer often presents in a cer
slow and vague manner b. Pancreatic cancer often presents in a
c. Severe pain is an early feature of this slow and vague manner
disease d. There are no specific blood tests to
d. There are no specific blood tests to diagnose pancreatic cancer
diagnose pancreatic cancer f. Chronic pancreatitis predisposes a pa-
e. Chemotherapy is the treatment of tient to pancreatic cancer
choice for pancreatic cancer
f. Chronic pancreatitis predisposes a pa-
tient to pancreatic cancer

21 / 31
Iggy ch 58, 59 Quiz
Study online at https://quizlet.com/_9gkz85
The nurse detects an epigastric mass
while assessing a patient with acute
pancreatitis. The patient describes epi-
gastric pain that radiates to his back.
What does the nurse suspect? b. Pancreatic pseudocyst
a. Liver cirrhosis
b. Pancreatic pseudocyst
c. Gallstones
d. Chronic pancreatitis
The nurse is caring for a patient with
pancreatic cancer who had a Whipple
procedure. Which interventions and as-
sessments does the nurse implement?
a. Place the patient in semi-Fowler's po-
SATA.
sition
a. Place the patient in semi-Fowler's po-
b. Place the NG tube on intermittent suc-
sition
tion
b. Place the NG tube on intermittent suc-
d. Keep the patient NPO
tion
e. Check blood glucose often
c. Monitor NG drainage, which should be
f. Monitor for signs of hypovolemia to pre-
bile-tinged and contain blood
vent shock
d. Keep the patient NPO
e. Check blood glucose often
f. Monitor for signs of hypovolemia to
prevent shock
What is the most common and serious
complication after a Whipple procedure?
a. Diabetes mellitus
c. Fistula development
b. Wound infection
c. Fistula development
d. Bowel obstruction
Which are manifestations of pancreatic
cancer? SATA. b. Anorexia and weight loss
a. Light colored uring and dark colored c. Splenomegaly
stools d. Ascites
b. Anorexia and weight loss e. Leg or calf pain
c. Splenomegaly f. Weakness and fatigue
d. Ascites

22 / 31
Iggy ch 58, 59 Quiz
Study online at https://quizlet.com/_9gkz85
e. Leg or calf pain
f. Weakness and fatigue
The nurse is teaching a patient and
family ahow to prevent exacerbations
of chronic pancreatitis. Which teaching
point does the nurse include?
a. Moderation in the use of caffeinated
beverages b. Avoidance of alcohol and nicotine
b. Avoidance of alcohol and nicotine
c. Consume a bland, high-fat, low-pro-
tein diet
d. Regular exercise, emphasizing aero-
bic activites
The patient is to continue pancreatic en-
zyme replacement therapy (PERT) af-
ter discharge. Which statement indicates
that the patient understands teaching
about this therapy?
a. "I will take the enzymes before meals
b. "I will take the enzymes after I take my
with a full glass of water."
ranitadine."
b. "I will take the enzymes after I take my
ranitadine."
c. "I will mix enzymes with chopped
meat."
d. "I will chew the capsules before swal-
lowing the enzymes."
Which are potential cardiovascular com-
plications for a patient after surgery for a
Whipple procedure? SATA.
a. Thrombophlebitis
a. Thrombophlebitis
c. Myocardial infarction
b. Pulmonary embolism
d. Heart failure
c. Myocardial infarction
f. Hemorrhage at anastomosis sites with
d. Heart failure
hypovolemia
e. Renal failure
f. Hemorrhage at anastomosis sites with
hypovolemia
Which abdominal laboratory findings are
cardinal findings in acute pancreatitis?
23 / 31
Iggy ch 58, 59 Quiz
Study online at https://quizlet.com/_9gkz85
SATA.
a. Elevated serum lipase
a. Elevated serum lipase
b. Increased serum amylase
b. Increased serum amylase
c. Decreased serum trypsin
d. Elevated serum elastase
d. Elevated serum elastase
f. Elevated bilirubin
e. Decreased serum glucose
f. Elevated bilirubin
Which are advantages of minimally inva-
sice surgery (MIS) laparoscopic chole-
cystectomy? SATA.
a. Complications are uncommon
a. Complications are uncommon
b. The mortality is similar to traditional
c. Patients recover more rapidly
cholecystectomy
d. Postoperative pain is less severe
c. Patients recover more rapidly
e. Bile duct injuries are rare
d. Postoperative pain is less severe
e. Bile duct injuries are rare
f. IV antibiotics are never needed be-
cause of decreased infection rates
What is one of the main advantages
of cholecystectomy by the natural ori-
fice transluminal endoscopic surgery
(NOTES) procedure?
a. Very small visible incisions
c. No visible incision lines
b. Jackson-Pratt drain removes excess
fluid
c. No visible incision lines
d. Resumption of normal activities the
day of surgery
Which is a key feature of pancreatic can-
cer?
a. Anorexia
a. Anorexia
b. Weight gain
c. Pale urine
d. Dark-colored stools
The patient with acute recrotizing pan-
creatitis experiences a temperature
spike to 104F (40C). What does the
nurse suspect?
24 / 31
Iggy ch 58, 59 Quiz
Study online at https://quizlet.com/_9gkz85
a. Pancreatitic pseudocyst
b. Pancreatic abscess
b. Pancreatic abscess
c. Chronic pancreatitis
d. Pancreatic cancer
The nurse is collaborating with the dieti-
cian to provide diet teaching for a patient
with chronic pancreatitis and his family.
Which are important teaching points for
htis teaching plan? SATA.
a. The patient will need increased calorie a. The patient will need increased calorie
intake (4000-6000) per day to maintain intake (4000-6000) per day to maintain
weight weight
b. Be sure to include foods that are high c. Alcohol use should be avoided
in fat because they are essential for heal- d. Provide a bland diet with frequent
ing meals
c. Alcohol use should be avoided e. Avoid irritating substances such as
d. Provide a bland diet with frequent caffeinated beverages which stimulate
meals the GI system
e. Avoid irritating substances such as
caffeinated beverages which stimulate
the GI system
f. Add rich foods to the diet to help meet
the caloric requirements
A nurse is preparing a teaching plan for
a client with a historu of cholelithiasis.
Which information about why the igen-
stion of fatty foods will comfort should the
nurse include in the teaching plan?
a. Fatty foods are hard to digest
b. Bile flow into the intestine in obstructed
b. Bile flow into the intestine in obstruct-
ed
c. The liver is manufacturing inadequate
bile
d. There is inadequate closure of the am-
pulla of Vater
A nurse is caring for a client with
cholelithiasis and obstructive jaundice.
When assessing this client, the nurse
25 / 31
Iggy ch 58, 59 Quiz
Study online at https://quizlet.com/_9gkz85
should be alert for which common clini-
cal indicators associated with these con-
ditions? SATA.
a. Ecchymosis
a. Ecchymosis
b. Yellow sclera
b. Yellow sclera
e. Pain in right upper quadrant
c. Dark brown stool
d. Straw-colored urine
e. Pain in right upper quadrant
For which clinical indicators should the
nurse monitor for when caring for a client
with cholelithiasis and obstructive jaun-
dice? SATA. a. Dark urine
a. Dark urine b. Yellow skin
b. Yellow skin d. Clay-colored stool
c. Pain on urination
d. Clay-colored stool
e. Coffee-ground vomitus
A nurse is caring for a client with a di-
agnosis of acute pancreatitis and alco-
holism. The client asks, "What does my
drinking have to do with my diagnosis?"
What effect of alcohol should the nurse
include when responding?
a. Promotes the formation of calculi in
the cystic duct d. Increases enzyme secretion and pan-
b. Stimulates the pancreas to secrete creatic duct pressure that causes back-
more insulin than it can immediately pro- flow of enzymes into the pancreas
duce
c. Alters the composition of enzymes so
they are capable of damaging the pan-
creas
d. Increases enzyme secretion and pan-
creatic duct pressure that causes back-
flow of enzymes into the pancreas
A client is diagnosed with cancer of the
pancreas and is apprehensive and rest-
less. Which nursing action should be in-
cluded in the plan of care?
26 / 31
Iggy ch 58, 59 Quiz
Study online at https://quizlet.com/_9gkz85
a. Encouraging expression of concerns
b. Administering antibiotics as pre-
scribed
c. Teaching the importance of getting a. Encouraging expression of concerns
rest
d. Explaining that everything will be al-
right
A client is admitted with a tentative
diagnosis of pancreatitis. The medical
and nursing measures for this client are
aimed toward maintaining nutrition, pro-
moting rest, maintaining fluid and elec-
trolyte balance, and decreasing anxiety.
b. Administer analgesics
Which interventions should the nurse im-
c. Teach relaxation exercises
plement? SATA.
e. Monitor cardiac rate and rhythm
a. Provide a low-fat diet
b. Administer analgesics
c. Teach relaxation exercises
d. Encourage walking in the hall
e. Monitor cardiac rate and rhythm
f. Observe for signs of hyper calcemia
A client with chronic hepatic failure is
soon to be discharged from the hospital.
Which diet should the nurse encourage
the client to follow based on the health
care provider's order? c. Low-protein
a. High-fat
b. Low-calorie
c. Low-protein
d. High-sodium
For shich clinical indicator associated
with a complication of portal hyperten-
sion should the nurse assess the client?
a. Liver abscess d. Hemorrhage from esophageal varices
b. Intestinal obstruction
c. Perforation of the duodenum
d. Hemorrhage from esophageal varices

27 / 31
Iggy ch 58, 59 Quiz
Study online at https://quizlet.com/_9gkz85
A client with hepatic cirrhosis begins
to develop slurred speech, confusion,
drowsiness, and a flapping tremor. With
this evidence of impending hepatic
coma, which diet can the nurse expect
a. 20 g of protein, 2000 calories
will be ordered for this client?
a. 20 g of protein, 2000 calories
b. 70 g of protein, 1200 calories
c. 80 g of protein, 2500 calories
d. 100 g of protein, 1500 calories
A client is admitted to the hospital with
a diagnosis of cirrhosis of the liver. For
which classic signs of hepatic coma
should the nurse assess this client?
SATA. a. Mental confusion
a. Mental confusion d. Flapping hand tremors
b. Increased cholesterol
c. Brown-colored stools
d. Flapping hand tremors
e. Hyperactive deep tendon reflexes
A nurse is concerned that a client with
at diagnosis of cirrhosis of the liver may
experience the complication of hepatic
coma. For which clinical indicator should
the nurse assess this client? a. Icterus
a. Icterus
b. Urticaria
c. Uremic frost
d. Hemangioma
A nurse is caring for a client with cir-
rhosis of the liver. Which laboratory test
should the nurse monitor that, when ab-
normal, might identify a client who may
benefit from neomycin enemas? a. Ammonia level
a. Ammonia level
b. Culture and sensitivity
c. White blood cell count
d. Alanine aminotransferase level
28 / 31
Iggy ch 58, 59 Quiz
Study online at https://quizlet.com/_9gkz85
A client with cirrhosis of the liver has a
prolonged prothrombin time and a low
platelet count. A regular diet is ordered.
What should the nurse instruct the client
to do considering the client's condition?
d. Report signs of bleeding no matter
a Avoid foods high in vitamin K
how slight
b. Check the pulse several times a day
c. Drink a glass of milk when taking as-
pirin
d. Report signs of bleeding no matter
how slight
The night shift nurse has just finished
giving the RN team leader a report on
the six clients. Which client has the high-
est acuity level and is at greatest risk for
shock during the shift?
a. Ms. H (acute cholecystitis) f. Mr. R (acute pancreatitis)
b. Ms. D (bowel obstruction)
c. Ms. T (ulcerative colitis)
d. Mr. A (appendectomy)
e. Mr. K (PEG-tube)
f. Mr. R (acute pancreatitis)
Which clients would be best to assign to
the new RN? SATA.
a. Ms. H (acute cholecystitis)
a. Ms. H (acute cholecystitis)
b. Ms. D (bowel obstruction)
b. Ms. D (bowel obstruction)
c. Ms. T (ulcerative colitis)
d. Mr. A (appendectomy)
d. Mr. A (appendectomy)
e. Mr. K (PEG-tube)
f. Mr. R (acute pancreatitis)
Which tasks can be delegated to the
a. Assisting Ms. T with perineal care after
UAP? SATA.
diarrheal episodes
a. Assisting Ms. T with perineal care after
b. Measuring vital signs every 2 hours for
diarrheal episodes
Mr. R
b. Measuring vital signs every 2 hours for
c. Transporting Ms. H off the unit for a
Mr. R
procedure
c. Transporting Ms. H off the unit for a
d. Gently cleansing the nares around Ms.
procedure
29 / 31
Iggy ch 58, 59 Quiz
Study online at https://quizlet.com/_9gkz85
d. Gently cleansing the nares around Ms.
D's NG tube D's NG tube
e. Removing Mr. A's dressing f. Helping Mr. K to brush his teeth
f. Helping Mr. K to brush his teeth
Which reporting tasks are appropriate to
delegate to the UAP? SATA.
a. Reporting on the condition of Ms. T's
perineal area after application of oint-
c. Reporting if any of the clients indicate
ment
pain
b. Reporting the quality and color of NG
d. Reporting whether Mr. R's blood pres-
drainage for Ms. D
sure is below 100/60 mmHg
c. Reporting if any of the clients indicate
e. Reporting if Mr. A is seen leaving the
pain
unit to smoke a cigarette
d. Reporting whether Mr. R's blood pres-
f. Reporting that Mr. K's family has ques-
sure is below 100/60 mmHg
tions
e. Reporting if Mr. A is seen leaving the
unit to smoke a cigarette
f. Reporting that Mr. K's family has ques-
tions
The night nurse gives a brief and incom-
plete report. Which question should the
oncoming RN leader pose to the night
shift nurse to help determine the priority
actions for Ms. H who was admitted for
acute cholecystitis? b. "Is she going to have surgery or radi-
a. "What are her vital signs?" atiology this morning?"
b. "Is she going to have surgery or radi-
atiology this morning?"
c. "Is she still having pain?"
d. "Does she need any morning medica-
tions?"
The health care provider told Ms. H
(acute cholecystitis) that she would
probably need a laparoscopic chole-
cystectomy; however, the hepatobiliary
iminodiacetic acid (HIDA) scan and lab-
oratory resilts are still pending. Ms. H
asks, "What should I expect?" What is
30 / 31
Iggy ch 58, 59 Quiz
Study online at https://quizlet.com/_9gkz85
the best intervention at this point?
a. Describe the surgical procedure
b. Call the HCP to come speak with her
c. Provide some written material about c. Provide some written material about
gallbladder disease and options gallbladder disease and options
d. Ecplain general postoperative care,
such as coughing and deep breathing
exercises
Ms. H's (acute cholecystitis) HIDA scan
shows a decreased bile flow with gall-
bladder disease and obstruction. Be-
cause of the obstruction, the nurse is vig-
ilant for the complication of biliary colic.
What are the key signs and symptoms
that the nurse will watch for?
a. Rebound tenderness and a
d. Severe abdominal pain with tachycar-
sausage-shaped mass in the right upper
dia, pallor, diaphoresis, and prostration
quadrant
b. Flatulence, dyspepsia, and eructation
after eating or drinking
c. Right upper quadrant abdominal pain
that radiates to the right shoulder or
scapula
d. Severe abdominal pain with tachycar-
dia, pallor, diaphoresis, and prostration
All of these clients must receive their rou-
tine morning medications. Which client
should receive his or her medication
last?
d. Mr. K (PEG-tube)
a. Ms. H (acute cholecystitis)
b. Ms. D (bowel obstruction)
c. Ms. T (ulcerative colititis)
d. Mr. K (PEG-tube)

31 / 31
Ch. 49,
Study online at https://quizlet.com/_86stsg

A 48-year-old male patient screened for


ANS: C
diabetes at a clinic has a fasting plasma
The patient's impaired fasting glucose
glucose level of 120 mg/dL (6.7 mmol/L).
indicates prediabetes, and the patient
The nurse will plan to teach the patient
should be counseled about lifestyle
about
changes to prevent the development of
a. self-monitoring of blood glucose.
type 2 diabetes. The patient with predi-
b. using low doses of regular insulin.
abetes does not require insulin or oral
c. lifestyle changes to lower blood glu-
hypoglycemics for glucose control and
cose.
does not need to self-monitor blood glu-
d. effects of oral hypoglycemic medica-
cose.
tions.
ANS: B
A 34-year-old has a new diagnosis of Because many patients have some dia-
type 2 diabetes. The nurse will discuss betic retinopathy when they are first di-
the need to schedule a dilated eye exam agnosed with type 2 diabetes, a dilated
a. every 2 years. eye exam is recommended at the time
b. as soon as possible. of diagnosis and annually thereafter. Pa-
c. when the patient is 39 years old. tients with type 1 diabetes should have
d. within the first year after diagnosis. dilated eye exams starting 5 years after
they are diagnosed and then annually.
ANS: A
A 27-year-old patient admitted with di- Hypokalemia can lead to potentially fatal
abetic ketoacidosis (DKA) has a serum dysrhythmias such as ventricular tachy-
glucose level of 732 mg/dL and serum cardia and ventricular fibrillation, which
potassium level of 3.1 mEq/L. Which would be detected with electrocardio-
action prescribed by the health care gram (ECG) monitoring. Because potas-
provider should the nurse take first? sium must be infused over at least 1
a. Place the patient on a cardiac monitor. hour, the nurse should initiate cardiac
b. Administer IV potassium supple- monitoring before infusion of potassium.
ments. Insulin should not be administered with-
c. Obtain urine glucose and ketone lev- out cardiac monitoring because insulin
els. infusion will further decrease potassium
d. Start an insulin infusion at 0.1 levels. Urine glucose and ketone levels
units/kg/hr. are not urgently needed to manage the
patient's care.
A 26-year-old patient with diabetes rides
a bicycle to and from work every day.
Which site should the nurse teach the
1 / 55
Ch. 49,
Study online at https://quizlet.com/_86stsg
patient to administer the morning in- ANS: C
sulin? Patients should be taught not to adminis-
a. thigh. ter insulin into a site that will be exercised
b. buttock. because exercise will increase the rate
c. abdomen. of absorption. The thigh, buttock, and
d. upper arm. arm are all exercised by riding a bicycle.
A 38-year-old patient who has type 1 ANS: A
diabetes plans to swim laps daily at 1:00 The change in exercise will affect blood
PM. The clinic nurse will plan to teach the glucose, and the patient will need to
patient to monitor glucose carefully to determine
a. check glucose level before, during, the need for changes in diet and insulin
and after swimming. administration. Because exercise tends
b. delay eating the noon meal until after to decrease blood glucose, patients are
the swimming class. advised to eat before exercising. Increas-
c. increase the morning dose of neutral ing the morning NPH or timing the in-
protamine Hagedorn (NPH) insulin. sulin to peak during exercise may lead
d. time the morning insulin injection so to hypoglycemia, especially with the in-
that the peak occurs while swimming. creased exercise.
Which finding indicates a need to con-
tact the health care provider before
the nurse administers metformin (Glu-
cophage)? ANS: D
a. The patient's blood glucose level is The BUN indicates possible renal fail-
174 mg/dL. ure, and metformin should not be used
b. The patient has gained 2 lb (0.9 kg) in patients with renal failure. The other
since yesterday. findings are not contraindications to the
c. The patient is scheduled for a chest use of metformin.
x-ray in an hour.
d. The patient's blood urea nitrogen
(BUN) level is 52 mg/dL.
ANS: B
Which action should the nurse take after
Rebound hypoglycemia can occur after
a 36-year-old patient treated with intra-
glucagon administration, but having a
muscular glucagon for hypoglycemia re-
meal containing complex carbohydrates
gains consciousness?
plus protein and fat will help prevent
a. Assess the patient for symptoms of
hypoglycemia. Orange juice and nonfat
hyperglycemia.
milk will elevate blood glucose rapidly,
b. Give the patient a snack of peanut
but the cheese and crackers will stabilize
2 / 55
Ch. 49,
Study online at https://quizlet.com/_86stsg
blood glucose. Administration of IV glu-
butter and crackers.
cose might be used in patients who were
c. Have the patient drink a glass of or-
unable to take in nutrition orally. The pa-
ange juice or nonfat milk.
tient should be assessed for symptoms
d. Administer a continuous infusion of
of hypoglycemia after glucagon adminis-
5% dextrose for 24 hours.
tration.
A 26-year-old female with type 1 dia-
betes develops a sore throat and runny ANS: C
nose after caring for her sick toddler. The Infection and other stressors increase
patient calls the clinic for advice about blood glucose levels and the patient will
her symptoms and a blood glucose level need to test blood glucose frequently,
of 210 mg/dL despite taking her usual treat elevations appropriately with lispro
glargine (Lantus) and lispro (Humalog) insulin, and call the health care provider
insulin. The nurse advises the patient to if glucose levels continue to be elevat-
a. use only the lispro insulin until the ed. Discontinuing the glargine will con-
symptoms are resolved. tribute to hyperglycemia and may lead to
b. limit intake of calories until the glucose diabetic ketoacidosis (DKA). Decreasing
is less than 120 mg/dL. carbohydrate or caloric intake is not ap-
c. monitor blood glucose every 4 hours propriate because the patient will need
and notify the clinic if it continues to rise. more calories when ill. Glycosylated he-
d. decrease intake of carbohydrates un- moglobin testing is not used to evaluate
til glycosylated hemoglobin is less than short-term alterations in blood glucose.
7%.
A 32-year-old patient with diabetes is
starting on intensive insulin therapy. ANS: A
Which type of insulin will the nurse dis- Rapid- or short-acting insulin is used for
cuss using for mealtime coverage? mealtime coverage for patients receiving
a. Lispro (Humalog) intensive insulin therapy. NPH, glargine,
b. Glargine (Lantus) or detemir will be used as the basal in-
c. Detemir (Levemir) sulin.
d. NPH (Humulin N)
Which patient action indicates good
understanding of the nurse's teach- ANS: B
ing about administration of aspart (No- Cleaning the skin with soap and water or
voLog) insulin? with alcohol is acceptable. Insulin should
a. The patient avoids injecting the insulin not be frozen. The patient should leave
into the upper abdominal area. the syringe in place for about 5 sec-
b. The patient cleans the skin with soap
3 / 55
Ch. 49,
Study online at https://quizlet.com/_86stsg
and water before insulin administration.
c. The patient stores the insulin in
onds after injection to be sure that all
the freezer after administering the pre-
the insulin has been injected. The upper
scribed dose.
abdominal area is one of the preferred
d. The patient pushes the plunger down
areas for insulin injection.
while removing the syringe from the in-
jection site.
When comparing the pathophysiology of
type 1 and type 2 diabetes, which state-
ment would be correct for a patient with
type 2 diabetes who was admitted to the
hospital with pneumonia?

a. The patient must receive insulin ther-


apy to prevent the development of ke-
d. The patient may have sufficient en-
toacidosis.
dogenous insulin secretion to prevent
b. The patient has islet cell antibodies
ketosis but is at risk for development of
that have destroyed the ability of the pan-
hyperosmolar hyperglycemic syndrome.
creas to produce insulin.
c. The patient has minimal or absent en-
dogenous insulin secretion and requires
daily insulin injections.
d. The patient may have sufficient en-
dogenous insulin secretion to prevent
ketosis but is at risk for development of
hyperosmolar hyperglycemic syndrome.
Analyze the following diagnostic findings
for your patient with type 2 diabetes.
Which result will need further assess-
ment?
b. A1C 9%
a. BP 126/80 mm Hg
b. A1C 9%
c. FBG 130mg/dL (7.2 mmol/L)
d. LDL cholesterol 100 mg/dL (2.6
mmol/L)
Which statement by the patient with type
2 diabetes is accurate?
4 / 55
Ch. 49,
Study online at https://quizlet.com/_86stsg

a. "I am supposed to have a meal or


snack if I drink alcohol."
b. "I am not allowed to eat any sweets
because of my diabetes." a. "I am supposed to have a meal or
c. "I do not need to watch what I eat be- snack if I drink alcohol."
cause my diabetes is not the bad kind."
d. "The amount of fat in my diet is not im-
portant; it is just the carbohydrates that
raise my blood sugar."
You are caring for a patient with newly
diagnosed type 1 diabetes. What infor-
mation is essential to include in your pa-
tient teaching before discharge from the
hospital (select all that apply)? a. Insulin administration
d. Hypoglycemia prevention, symptoms,
a. Insulin administration and treatment
b. Elimination of sugar from diet
c. Need to reduce physical activity
d. Hypoglycemia prevention, symptoms,
and treatment
What is the priority action for the nurse
to take if the patient with type 2 diabetes
complains of blurred vision and irritabili-
ty?
c. Check the patients blood glucose lev-
a. Call the physician.
el.
b. Administer insulin as ordered.
c. Check the patients blood glucose lev-
el.
d. Assess for other neurologic symp-
toms.
A diabetic patient has a serum glucose
level of 824 mg/dL (45.7 mmol/L) and is
unresponsive. Following assessment of
c. rapid, deep respirations.
the patient, the nurse suspects diabetic
ketoacidosis rather than hyperosmolar
hyperglycemic syndrome based on the
5 / 55
Ch. 49,
Study online at https://quizlet.com/_86stsg
findings of

a. polyuria.
b. severe dehydration.
c. rapid, deep respirations.
d. decreased serum potassium.
Which are appropriate therapies for pa-
tients with diabetes mellitus (select all
that apply)?
a. Use of statins to treat dylipidemia
c. Use of ACE inhibitors to treat
a. Use of statins to treat dyslipidemia
nephropathy
b. Use of diuretics to treat nephropathy
d. Use of laser photocoagulation to treat
c. Use of ACE inhibitors to treat
retinopathy
nephropathy
d. Use of laser photocoagulation to treat
retinopathy
In addition to promoting the transport
of glucose from the blood into the cell,
d. Rationale: Insulin is an anabolic hor-
insulin also
mone, responsible for growth, repair, and
storage, and it facilitates movement of
a. enhances the breakdown of adipose
amino acids into cells, synthesis of pro-
tissue for energy.
tein, storage of glucose as glycogen, and
b. stimulates hepatic glycogenolysis and
deposition of triglycerides and lipids as
gluconeogenesis.
fat into adipose tissue. Glucagon is re-
c. prevents the transport of triglycerides
sponsible for hepatic glycogenolysis and
into adipose tissue.
gluconeogenesis, and fat is used for en-
d. accelerates the transport of amino
ergy when glucose levels are depleted.
acids into cells and their synthesis into
protein.
d. Rationale: Type 2 diabetes has a
Which of the following patients would a
strong genetic influence, and offspring of
nurse plan to teach how to prevent or
parents who both have type 2 diabetes
delay the development of diabetes?
have an increased chance of developing
it. Whereas type 1 diabetes is associat-
a. A 62 year-old obese white woman.
ed with genetic susceptibility related to
b. An obese 50 year-old Hispanic
human leukocyte antigens (HLAs), off-
woman.
spring of parents who both have type 1
c. A child whose father has type 1 dia-
diabetes have only a 6% to 10% chance
6 / 55
Ch. 49,
Study online at https://quizlet.com/_86stsg
of developing the disease. Lower risk fac-
betes. tors for type 2 diabetes include obesity;
d. A 34 year-old woman whose parents being a Native American, Hispanic, or
both have type 2 diabetes. African-American; and being 55 years or
older.
a. Rationale: Metabolic syndrome is
When caring for a patient with metabolic a cluster of abnormalities that in-
syndrome, the nurse gives the highest clude elevated insulin levels, elevat-
priority to teaching the patient about ed triglycerides and low-density lipopro-
teins (LDL), and decreased high-density
a. maintaining a normal weight. lipoproteins (HDL). These abnormalities
b. performing daily aerobic exercise. greatly increase the risk for cardiovas-
c. eliminating red mean from the diet. cular disease associated with diabetes
d. monitoring the blood glucose periodi- that can be prevented or delayed with
cally. weight loss. Exercise is also important,
but normal weight is most important.
During routine health screening, a pa-
tient is found to have a fasting plas- b,c. Rationale: The patient has one pri-
ma glucose (FPG) of 132 mg/dL (7.33. or test result that meets criteria for a
mmol/L). At a follow-up visit, a diagnostic diagnosis of diabetes, but on a subse-
of diabetes would be made based on quent day must again have results from
(select all that apply) on of the three tests that meet the cri-
teria for diabetes diagnosis. These cri-
a. glucosuria of 3+. teria include a fasting plasma glucose
b. an A1C of 7.5%. level of e126mg/dL (7.0 mmol/L), or A1C
c. a FPG of e126mg/dL (6.9 mmol/L). e6.5% or a 2 hour OGTT level e200mg/dL
d. random blood glucose of 126 mg/dL (11.1 mmol/L). Both the fasting plasma
(7.0 mmol/L). glucose (FPG) and A1C would confirm a
e. a 2-hour oral glucose tolerance test diagnosis of diabetes in this patient.
(OGTT) of 190 mg/dL (10.5 mmol/L).
c. Rationale: Impaired glucose tolerance
The nurse determines that a patient with exists when a 2-hour plasma glucose
a 2-hour OGTT of 152mg/dL has level is higher than normal but lower than
the level diagnostic for diabetes (i.e.,
a. diabetes. 140-199mg/dL). Impaired fasting glu-
b. impaired fasting glucose. cose exists when fasting glucose levels
c. impaired glucose tolerance. are greater than the normal of 100mg/dL
but less than the 126mg/dL diagnostic
7 / 55
Ch. 49,
Study online at https://quizlet.com/_86stsg
d. elevated glycosylated hemoglobin of diabetes. Both conditions represent a
(Hb) condition known as pre diabetes.
When teaching the patient with diabetes c. Rationale: U100 insulin must be used
about insulin administration, the nurse with a U100 syringe, but for those us-
instructs the patient to ing low doses of insulin, syringes are
available that have increments of 1 unit
a. pull back on the plunger after inserting instead of 2 units. Errors can be made
the needle to check for blood. in dosing if patients switch back and
b. clean the skin at the injection site with forth between different sizes of syringes.
an alcohol swab before each injection. Aspiration before injection of the insulin
c. consistently use the same size of the is not recommended, nor is the use of
appropriate strength insulin syringe to alcohol to clean the skin. Because the
avoid dosing errors. rate of peak serum concentrations varies
d. rotate injection sites from arms to with the site selected for injection, injec-
thighs to abdomen with each injection to tions should be rotated within a particu-
prevent lipodystrophies. lar area, such as the abdomen.
A patient with type 1 diabetes uses
20U of 70/30 neutral protamine Hage-
dorn (NPH/regular) in the morning and
b. Rationale: A split-mixed dose of in-
at 6:00pm. When teaching the patient
sulin requires that the patient adhere to
about this regimen, the nurse stresses
a set meal pattern to provide glucose for
that
the action of the insulins, and a bedtime
snack is usually required when patients
a. hypoglycemia is most likely to occur
take a long-acting insulin late in the day
before the noon meal.
to prevent nocturnal hypoglycemia. Hy-
b. a set meal pattern with a bedtime
poglycemia is most likely to occur with
snack is necessary to prevent hypo-
this dose late in the afternoon and during
glycemia.
the night. When premixed formulas are
c. flexibility in food intake is possible be-
used, flexible dosing based on glucose
cause insulin is available 24 hours/day.
levels is not recommended.
d. pre-meal glucose checks are required
to determine needed changes in daily
dosing.
Lispro insulin (Humalog) with NPH in-
d. Rationale: Lispro is a rapid-acting in-
sulin is ordered for a patient with newly
sulin that has an onset of action of 5
diagnosed type 1 diabetes. The nurse
to 15 minutes and should be injected at
knows that when lispro insulin is used, it
the time of the meal to within 15 minutes
should be administered
8 / 55
Ch. 49,
Study online at https://quizlet.com/_86stsg

a. only once a day of eating. Regular insulin is short acting


b. 1 hour before meals with an onset of action in 30 to 60 min-
c. 30-45 minutes before meals utes following administration and should
d. at mealtime or within 15 minutes of be given 30 to 45 minutes before meals.
meals
A diabetic patient is learning to mix reg-
ular insulin and NPH insulin in the same
a. Rationale: When mixing regular with
syringe. The nurse determines that addi-
a longer-acting insulin, regular insulin
tional teaching is needed when the pa-
should always be drawn in the syringe
tient
first to prevent contamination of the reg-
ular insulin vial with longer-acting insulin
a. withdraws the NPH dose in the syringe
additives. Air is added in the neutral pro-
first
tamine Hagedorn (NPH) vial; then air is
b. injects air equal to the NPH dose into
added to the regular vial, and the regu-
the NPH vial first.
lar insulin is withdrawn, bubbles are re-
c. removes any air bubbles after with-
moved, and the dose of NPH is with-
drawing the first insulin.
drawn.
d. adds air equal to the insulin dose into
the regular vial and withdraws the dose.
The following interventions are planned
for a diabetic patient. Which intervention
can the nurse delegate to nursing assis-
b. Rationale: Checking the temperature
tive personnel (NAP)?
of the bath water is part of assisting
with activities of daily living (ADLs) and
a. Discuss complications of diabetes.
within the scope of care for the nursing
b. Check that the bath water is not too
assistive personnel (NAP). Discussion of
hot.
complications, teaching, and assessing
c. Check the patient's technique for
learning are appropriate for RNs.
drawing up insulin.
d. Teach the patient to use the glucome-
ter for in-home glucose monitoring.
The home care nurse should intervene
to correct a patient whose insulin admin-
d. Rationale: Insulin glargine (Lantus), a
istration includes
long-acting insulin that is continuously
released with no peak of action, cannot
a. warming a pre filled refrigerated sy-
ringe in the hands before administration.
9 / 55
Ch. 49,
Study online at https://quizlet.com/_86stsg
b. storing syringes pre filled with NPH
and regular insulin needle-up in the re-
be diluted or mixed with any other in-
frigerator.
sulin or solution. Mixed insulins should
c. placing the insulin bottle currently in
be stored needle-up in the refrigerator
use in a small container on the bathroom
and warmed before administration. Cur-
countertop.
rently used bottles of insulin can be kept
d. mixing an evening dose of regular in-
at room temperature.
sulin with insulin glargine in one syringe
for administration.
The major advantage of using an insulin
pump is that
a. Rationale: Insulin pumps provide tight
glycemic control by continuous subcuta-
a. tight glycemic control can be main-
neous insulin infusion based on the pa-
tained.
tient's basal profile, with bolus doses at
b. errors in insulin dosing are less likely
mealtime at the patient's discretion. Er-
to happen.
rors in insulin dosing and complications
c. complications of insulin therapy are
of insulin therapy are still potential risks
prevented.
with insulin pumps.
d. frequent blood glucose monitoring is
unnecessary.
A patient taking insulin has recorded
fasting glucose levels above 200mg/dL
(11.1 mmol/L) on awakening for the last c. Rationale: The patient's elevated glu-
five mornings. The nurse advises the pa- cose on arising may be the result of
tient to either dawn phenomenon or Somogyi
effect, and the best way to determine
a. increase the evening insulin dose to whether the patient needs more or less
prevent the dawn phenomenon. insulin is by monitoring the glucose at
b. use a single dose insulin regimen with bedtime, between 2:00 and 4:00 am, and
an intermediate-acting insulin. on arising. If predawn levels are below
c. monitor the glucose level at bedtime, 60mg/dL, the insulin dose should be re-
between 2:00 and 4:00 am, and on aris- duced, but if the 2:00 to 4:00 am blood
ing. glucose is high, the insulin should be
d. decrease the evening insulin dosage increased.
to prevent night hypoglycemia and the
Somogyi effect.
The nurse is assessing a newly admitted
diabetic patient. Which of these observa-
10 / 55
Ch. 49,
Study online at https://quizlet.com/_86stsg
tions should be addressed as a priority
by the nurse?
c. Rationale: Rapid deep respirations
are symptoms of diabetic ketoacido-
a. Bilateral numbness of both hands
sis (DKA). Stage II ulcers and bilateral
b. Stage II pressure ulcer on the right
numbness are chronic complications of
heel
diabetes. The lumps and dents on the
c. Rapid respirations with deep inspira-
abdomen indicate a need to teach the
tion
patient about site rotation.
d. Areas of lumps and dents on the ab-
domen.
a. Rationale: The body requires food
at regularly spaced intervals through-
out the day, and omission or delay of
In nutritional management of all types of
meals can result in hypoglycemia, espe-
diabetes, it is important for the patient to
cially for the patient taking insulin or oral
hypoglycemic agents. Weight loss may
a. eat regular meals at regular times.
be recommended in type 2 diabetes if
b. restrict calories to promote moderate
the individual is overweight, but many
weight loss.
patients with type 1 diabetes are thin
c. eliminate sucrose and other simple
and require an increase in caloric intake.
sugars from the diet.
Fewer than 7% of total calories should be
d. limit saturated fat intake to 30% of
from saturated fats, and simple sugars
dietary intake.
should be limited, but moderate amounts
can used if counted as a part of total
carbohydrate intake.
b. Rationale: Maintenance of as
near-normal blood glucose levels as
Goals of nutritional therapy for the pa- possible and a achievement of optimal
tient with type 2 diabetes include main- serum lipid levels with dietary modifi-
tenance of cation are believed to be the most im-
portant factors in preventing both short-
a. ideal body weight. and long-term complications of diabetes.
b. normal serum glucose and lipid levels. There is no specific "diabetic diet," and
c. a special diet using diabetic foods. use of dietetic foods is not necessary
d. five small meals per day with a bed- for diabetes control. Most diabetics eat
time snack. three meals a day, and some require a
bedtime snack for control of nighttime hy-
poglycemia. A reasonable weight, which

11 / 55
Ch. 49,
Study online at https://quizlet.com/_86stsg
may or may not be an ideal body weight,
is also a goal of nutritional therapy.
To prevent hyperglycemia or hypo-
b. Rationale: During exercise, a diabetic
glycemia with exercise, the nurse teach-
person needs both adequate glucose to
es the patient using glucose-lowering
prevent exercise-induced hypoglycemia
agents that exercise should be undertak-
and adequate insulin because conter-
en
regulatory hormones are produced dur-
ing the stress of exercise and may cause
a. only after a 10- to 15-g carbohydrate
hyperglycemia. Exercise after meals is
snack is eaten.
best, but a 10- to 15-g carbohydrate
b. about 1 hour after a eating, when
snack may be taken if exercise is per-
blood glucose levels are rising.
formed before meals or is prolonged.
c. when glucose monitoring reveals that
Blood glucose levels should be moni-
the blood glucose is in the normal range.
tored before, during, and after exercise
d. when blood glucose levels are high
to determine the effect of exercise on the
because exercise always has a hypo-
levels.
glycemic effect.
The nurse assesses the diabetic pa-
tient's technique of self-monitoring of
c. Rationale: Cleaning the puncture site
blood glucose (SMBG) 3 months after
with alcohol is not necessary and may in-
initial instruction. An error in the perfor-
terfere with test results and lead to drying
mance of SMBG noted by the nurse that
and splitting of the fingertips. Washing
requires intervention is
the hands with warm water is adequate
cleaning and promotes blood flow to the
a. doing the SMBG before and after ex-
fingers. Blood flow is also increase by
ercising.
holding the hand down. Puncture on the
b. puncturing the finger on the side of the
side of the finger pad are less painful.
finger pad.
Self-monitored blood glucose (SMBG)
c. cleaning the puncture site with alcohol
should be performed before and after
before the puncture.
exercise.
d. holding the hand down for a few min-
utes before the puncture.
A nurse working in an outpatient clinic b. Rationale: The American Diabetes As-
plans a screening program for diabetes. sociation recommends that testing for
Recommendations for screening would type 2 diabetes with a FPG should be
include considered for all individuals at the age
of 45 and above and, if normal, repeated
a. OGTT fro all minority populations every 3 years. Testing for immune mark-
12 / 55
Ch. 49,
Study online at https://quizlet.com/_86stsg
every year. ers of type 1 diabetes is not recommend-
b. FPG for all individuals at age 45 and ed. Testing of a younger age or more
then every 3 years. frequently should be done for members
c. testing all people under the age of 21 of a high-risk ethnic population, includ-
for islet cell antibodies. ing African Americans, Hispanics, Native
d. testing for type 2 diabetes only in over- Americans, Asians Americans, and Pa-
weight or obese individuals. cific Islanders.
a. Rationale: During minor illnesses, the
A patient with diabetes calls the clinic be- patient with diabetes should continue
cause she is experiencing nausea and drug therapy and food intake. Insulin
flulike symptoms. The nurse advises the is important because counter regulato-
patient to ry hormones may raise blood glucose
during the stress of illness, and food or
a. administer the usual insulin dosage. a carbohydrate liquid substitution is im-
b. hold fluid intake until the nausea sub- portant because during illness the body
sides. requires extra energy to deal with the
c. come to the clinic immediately for eval- stress of the illness. Blood glucose mon-
uation and treatment. itoring should be done every 4 hour,
d. monitor the blood glucose every 1 to and the health care provider should be
2 hours and call if the glucose rises over notified if the level is >240mg/dL (13.9
150 mg/dL (8.3 mmol/L) mmol/L) or if fever, ketonuria, or nausea
and vomiting occur.
Ketoacidosis occurs as a complication of
diabetes when
c. Rationale: When insulin is insufficient
a. illnesses causing nausea and vomit- and glucose cannot be used for cellular
ing lead to bicarbonate loss with body energy, the body releasees and breaks
fluids. down stored fats and protein to meet en-
b. the glucose level becomes so high ergy needs. Free fatty acids from stored
that osmotic diuresis promotes fluid and triglycerides are released and metabo-
electrolyte loss. lized in the liver in such large quanti-
c. an insulin deficit causes the body to ties that ketones are formed. Ketones
metabolize large amounts of fatty acids are acidic and alter the pH of the blood,
rather than glucose for energy. causing acidosis. Osmotic diuresis oc-
d. the patient skips meals after taking curs as a result of loss of both glucose
insulin, leading to rapid metabolism of and ketones in the urine.
glucose and breakdown of fats for ener-
gy.

13 / 55
Ch. 49,
Study online at https://quizlet.com/_86stsg
d. Rationale: If a diabetic patient is
unconscious, immediate treatment for
A diabetic patient is found unconscious
hypoglycemia must be given to pre-
at home, and a family member calls
vent brain damage, and IM or subcuta-
the clinic. After determining that no glu-
neous administration of 1mg of glucagon
cometer is available, the nurse advise
should be done. If that unconscious-
the family member to
ness has another cause, such as ke-
tosis, the rise in glucose cause by the
a. try to around the patient to drink some
glucagon is not as dangerous as the
orange juice.
low glucose level. Following administra-
b administer 10 U or regular insulin sub-
tion of the glucagon, the patient should
cutaneously.
be transported to a medical facility for
c. call for an ambulance to transport the
further treatment and evaluation. Insulin
patient to a medical facility.
is contraindicated without knowledge of
d. administer glucagon 1mg intramuscu-
the patient's glucose level, and oral car-
larly (IM) or subcutaneously.
bohydrate cannot be given when pa-
tients are unconscious.
Two days following a self-managed hy-
poglycemic episode at home, the pa-
tient tells the nurse that his blood glu-
cose levels since the episode have been
between 80 and 90mg/dL. The best re- a. Rationale: Blood glucose levels of 80
sponse by the nurse is, to 90mg/dL (4.5-5 mmol/L) are within
the normal range and are desired in
a. "That is a good range for your glucose the patient with diabetes, even follow-
levels." ing a recent hypoglycemic episode. Hy-
b. "You should call your health care poglycemia is often caused by a single
provider because you need to have your event, such as skipping a meal or taking
insulin increased." too much insulin or vigorous exercise;
c. "That level is too low in view of your once corrected, normal control should
recent hypoglycemia, and you should in- be maintained.
crease your food intake."
d. "You should only take half your insulin
dosage for the next few days to get your
glucose level back to normal."
In diabets, atherosclerotic disease af- b. Rationale: The development of ather-
fecting the cerebrovascular, cardiovas- osclerotic vessel disease seems to be
cular, and peripheral vascular systems promoted by the altered lipid metabo-

14 / 55
Ch. 49,
Study online at https://quizlet.com/_86stsg

lism common to diabetes, and although


a. can be prevent by tight glucose con-
tight glucose control may help delay the
trol.
process, it does not prevent it completely.
b. occurs with a higher frequency and
Atherosclerosis in diabetic patients does
earlier onset than in the nondiabetic pop-
respond somewhat to a reduction in gen-
ulation.
eral risk factors, as it does in nondiabet-
c. is cause by the hyperinsulinemia relat-
ics, and reduction in fat intake, control
ed to insulin resistance common in type
of hypertension, abstention from smok-
2 diabetes.
ing, maintenance of normal weight, and
d. cannot be modified by reduction of risk
regular exercise should be carried out by
factors such as smoking, obesity, and
diabetic patients.
high fat intake.
Following the teaching of foot care to
a diabetic patient, the nurse determines
that additional instruction is needed
d. Rationale: Complete or partial loss of
when the patient says,
sensitivity of the feet is common with
peripheral neuropathy of diabetes, and
a. "I should wash my feet daily with soap
diabetics may suffer foot injury and ul-
and warm water."
ceration without ever having pain. Feet
b. "I should always wear shoes to protect
must be inspected during daily care for
my feet from injury."
any cuts, blisters, swelling, or reddened
c. "If my feet are cold, I should wear
areas.
socks instead of using a heating pad."
d. "I'll know if I have sores or lesions on
my feet because they will be painful."
a. Rationale: Because the clinical mani-
A 72-year-old woman is diagnosed with
festations of long-term complications of
diabetes. The nurse recognizes that
diabetes take 10 to 20 years to devel-
management of diabetes in the older
op, and because tight glucose control in
adult
the older patient is associated with an
increase frequency of hypoglycemia, the
a. does not require as tight glucose con-
goals for glycemic control are not as rigid
trol as in younger diabetics.
as in the younger population. Treatment
b. is usually not treated unless the pa-
is indicated, and insulin may be used
tient becomes severely hyperglycemic.
if the patient does not respond to oral
c. does not include treatment with insulin
agents. The patient's needs, rather than
because of limited dexterity and vision.
age, determine the responsibility of other
d. usually requires that a younger family
in care.

15 / 55
Ch. 49,
Study online at https://quizlet.com/_86stsg
member be responsible for care of the
patient.
A client is diagnosed with type 2 dia-
betes mellitus. The nurse is aware that
which statement is true?

a. Client is most likely a teenager.


c. Heredity is a major causative factor.
b. Client is most likely a child younger
than 10 years.
c. Heredity is a major causative factor.
d. Viral infections contribute most to dis-
ease development.
Antidiabetic drugs are designed to con-
trol signs and symptoms of diabetes
mellitus. The nurse primarily expects a
decrease in which?
a. Blood glucose
a. Blood glucose
b. Fat metabolism
c. Glycogen storage
d. Protein mobilization
A client is to receive insulin before break-
fast, and the time of breakfast tray de-
livery is variable. The nurse knows that
which insulin should not be administered
until the breakfast tray has arrived and
the client is ready to eat? b. lispro (Humalog)

a. Humulin N
b. lispro (Humalog)
c. glargine (Lantus)
d. Humulin R
A client is receiving a daily dose of Hu-
mulin N insulin at 7:30 am. The nurse
expects the peak effect of this drug to
c. 5:00 pm
occur at which time?

a. 8:15 am
16 / 55
Ch. 49,
Study online at https://quizlet.com/_86stsg
b. 10:30 am
c. 5:00 pm
d. 11:00 pm
A nurse ho is teaching a client how
to recognize symptoms of hypoglycemia
should include which symptoms in the
teaching? (Select all that apply.)
a. Headache
a. Headache b. Nervousness
b. Nervousness d. Sweating
c. Bradycardia
d. Sweating
e. Thirst
f. Sweet breath odor
A client is newly diagnosed with type 1
diabetes mellitus and requires daily in-
sulin injections. Which instruction should
the nurse include in the teaching of in-
sulin administration?

a. Teach the family members to adminis-


ter glucagon by injection if the client has b. Instruct the client about the necessi-
a hyperglycemic reaction. ty for compliance with prescribed insulin
b. Instruct the client about the necessity therapy.
for compliance with prescribed insulin
therapy.
c. Teach the client that hypoglycemic re-
actions more likely to occur at the onset
of action time.
d. Instruct the client in the care of insulin
container and syringe handling.
What are the major symptoms that char-
acterize diabetes? (Select all that apply.)
a. polyuria
b. polyphagia
a. polyuria
d. polydipsia
b. polyphagia
c. polyposia

17 / 55
Ch. 49,
Study online at https://quizlet.com/_86stsg
d. polydipsia
e. polyrrhea
What is the rationale for rotation of in-
sulin injection sites?

a. prevents polyuria d. prevents lipodystrophy


b. prevents rejection of insulin
c. prevents an allergic reaction
d. prevents lipodystrophy
What is the only type of insulin that may
be administered IV?

a. NPH d. Regular
b. Detemir
c. Lantus
d. Regular
Which clinical manifestations may be
seen in a client experiencing a hypo-
glycemic (insulin) reaction? (Select all
that apply.) a. headache
b. nervousness
a. headache c. tremor
b. nervousness d. excessive perspiration
c. tremor e. tachycardia
d. excessive perspiration
e. tachycardia
f. abdominal pain
Which clinical manifestations may be
seen in a client experiencing diabetic ke-
toacidosis (hyperglycemia)? (Select all
that apply.) a. thirst
b. polyuria
a. thirst d. Kussmaul's sign
b. polyuria e. dry mucous membranes
c. bradycardia f. fruity breath odor
d. Kussmaul's sign
e. dry mucous membranes
f. fruity breath odor
18 / 55
Ch. 49,
Study online at https://quizlet.com/_86stsg

Which information should be included in


health teaching for clients taking insulin?
(Select all that apply.)

a. Recognize signs of hypoglycemic re-


a. Recognize signs of hypoglycemic re-
action.
action.
b. Adhere to the prescribed diet.
b. Adhere to the prescribed diet.
c. Take insulin as prescribed.
c. Take insulin as prescribed.
d. Monitor blood glucose level.
d. Monitor blood glucose level.
e. Be sure to exercise.
f. Keep appointments with health care
provider.
g. Alter insulin dose based on how you're
feeling.
In addition to promoting the transport of
glucose from the blood into the cell, what
does insulin do?
a. enhances the breakdown of adipose
tissue for energy
d. accelerates the transport of amino
b. stimulates hepatic glycogenolysis and
acids into cells and their synthesis into
gluconeogenesis
protein
c. prevents the transport of triglycerides
into adipose tissue
d. accelerates the transport of amino
acids into cells and their synthesis into
protein
The nurse is teaching the patient with
prediabetes ways to prevent or delay the
development of type two diabetes. What
information should be included (select
all that apply).
a. maintain a healthy weight
a. maintain a healthy weight
e. monitor for polyuria, polyphagia, and
b. exercise for 60 minutes each day
polydipsia
c. have BP checked regularly
d. assess for visual changes on monthly
basis
e. monitor for polyuria, polyphagia, and
polydipsia
19 / 55
Ch. 49,
Study online at https://quizlet.com/_86stsg
In type 1 diabetes there is an osmotic
effect of glucose when insulin deficiency
prevents the use of glucose for energy.
Which classic symptom is caused by the
osmotic effect of glucose? b. polydipsia
a. fatigue
b. polydipsia
c. polyphagia
d. recurrent infections
Which patient should the nurse plan to
teach how to prevent or delay the devel-
opment of diabetes?
a. an obese 50 year old hispanic woman
b. a child whose father has type 1 dia-
c. a 34 year old woman whose parents
betes
both have type 2 diabetes
c. a 34 year old woman whose parents
both have type 2 diabetes
d. a 12 year old boy whose father has
maturity onset diabetes of the young
(MODY)
A patient with Type 1 diabetes uses 20
U of 70/30 neutral protamine Hagedorn
(NPH/regular) in the morning and at 6
pm. When teaching the patient about this
regimen, what should the nurse empha-
size?
a. hypoglycemia is most likely to occur
c. a set meal pattern with a bedtime
before the noon meal
snack is necessary to prevent hypo-
b. flexibility in food intake is possible be-
glycemia
cause insulin is available 24 hours a day
c. a set meal pattern with a bedtime
snack is necessary to prevent hypo-
glycemia
d. premeal glucose checks are required
to determine needed changes in daily
dosing
A patient with diabetes is learning to
mix regular insulin and NPH insulin in
20 / 55
Ch. 49,
Study online at https://quizlet.com/_86stsg
the same syringe. The nurse determines
that additional teaching is needed when
the patient does what?
a. withdraws the NPH dose into the sy-
ringe first
a. withdraws the NPH dose into the sy-
b. infects air equal to the NPH dose into
ringe first
the NPH vial first
c. removes any air bubbles after with-
drawing the first insulin
d. adds air equal to the insulin dose into
the regular vial and withdraws the dose
when teaching the patient with Type 1
diabetes, what should the nurse empha-
size as the major disadvantage of using
an insulin pump?
a. tight glycemic control can be main-
tained a. tight glycemic control can be main-
b. errors in insulin dosing are less likely tained
to occur
c. complications of insulin therapy are
prevented
d. frequent blood glucose monitoring is
unnecessary
A patient taking insulin has recorded
fasting glucose levels above 200 mg/dL
(11.1 mmol/L) on awakening for the last
five mornings. What should the nurse
advise the patient to do first?
a. increase the evening insulin dose to
prevent the dawn phenomenon c. monitor the glucose level at bedtime,
b. use a single-dose insulin regimen with between 2 AM and 4 AM, and on arising
an intermediate acting insuling
c. monitor the glucose level at bedtime,
between 2 AM and 4 AM, and on arising
d. decrease the evening insulin dosage
to prevent night hypoglycemia and the
Somogyi effect

21 / 55
Ch. 49,
Study online at https://quizlet.com/_86stsg
Which class of oral glucose-lowering
agents is most commonly used for peo-
ple with type 2 diabetes because is re-
duces hepatic glucose production and
enhances tissue uptake of glucose? b. biguanide
a. insulin
b. biguanide
c. meglitinide
d. sulfonylurea
The patient with type 2 diabetes is being
put on acarbose (precose) and wants to
know why she is taking it. What should
the nurse include in this patients teach-
ing (select all that apply) a. take it with the first bite of each meal
a. take it with the first bite of each meal d. effectiveness is measured by 2 hour
b. it is not used in patients with HF postprandial glucose
c. endogenous glucose production is de- e. it delays glucose absorption from the
creased GI tract
d. effectiveness is measured by 2 hour
postprandial glucose
e. it delays glucose absorption from the
GI tract
individualized nutrition therapy for pa-
tients using conventional, fixed insulin
regimens should include teaching the
patient to
a. eat regular meals at regular times
b. restrict calories to promote moderate a. eat regular meals at regular times
weight loss
c. eliminate sucrose and other simple
sugars from the diet
d. limit saturated fat intake to 30% of
dietary calorie intake
What should the goals of nutrition thera-
py for the patient with type two diabetes
include? b. normal serum glucose and lipid levels
a. ideal body weight
b. normal serum glucose and lipid levels
22 / 55
Ch. 49,
Study online at https://quizlet.com/_86stsg
c. a special diabetic diet using dietetic
foods
d. five small meals per day with a bed-
time snack
A patient with diabetes calls the clinic be-
cause she is experiencing nausea and
flu like symptoms. Which advice from the
nurse will be the best for this patient?
a. administer the usual insulin dosage
b. hold fluid intake until the nausea sub-
a. administer the usual insulin dosage
sides
c. come to the clinic immediately for eval-
uation and treatment
d. monitor the blood glucose everyone
one to two hours and call if it rises over
150 mg/dL (8.3 mmol/L)
The nurse should observe the patient for
symptoms of ketoacidosis when
a. illness causing nausea and vomiting
lead to bicarbonate loss with body fluids
b. glucose levels become so high that os-
motic diuresis promotes fluid and elec-
c. an insulin deficit causes the body to
trolyte loss
metabolize large amounts of fatty acids
c. an insulin deficit causes the body to
rather than glucose for energy
metabolize large amounts of fatty acids
rather than glucose for energy
d. the patient skips meals after taking
insulin, leading to rapid metabolism of
glucose and breakdown of fats for ener-
gy
What are manifestations of DKA (select
all that apply) a. thirst
a. thirst b. ketonuria
b. ketonuria c. dehydration
c. dehydration d. metabolic acidosis
d. metabolic acidosis e. Kussmaul respirations
e. Kussmaul respirations f. sweet, fruity breath odor
f. sweet, fruity breath odor
23 / 55
Ch. 49,
Study online at https://quizlet.com/_86stsg
What describes the primary difference
in treatment of DKA and hyperosmolar
hyperglycemia syndrome (HHS)
a. DKA requires administration of bicar-
bonate to correct acidosis
b. potassium replacement is not neces- c. HHS requires greater fluid replace-
sary in management of HHS ment to correct the dehydration
c. HHS requires greater fluid replace-
ment to correct the dehydration
d. administration of glucose is withheld
in HHS until the blood glucose reaches
a normal level
The patient with newly diagnosed di-
abetes is displaying shakiness, con-
fusion, irritability, and slurred speech.
What should the nurse suspect is hap-
pening c. hypoglycemia
a. DKA
b. HHS
c. hypoglycemia
d. hyperglycemia
the patient with diabetes has a blood
glucose level of 248 mg/dL. Which man-
ifestations in the patient would the nurse
understand as being related to this blood
a. headache
glucose level? (select all that apply)
c. abdominal cramps
a. headache
e. increase in urination
b. unsteady gait
f. weakness and fatigue
c. abdominal cramps
d. emotional changes
e. increase in urination
f. weakness and fatigue
what disorders and diseases are related
to macrovascular complications of dia-
betes? select all that apply
a. chronic kidney disease
b. coronary artery disease
c. microaneurysms and destruction or
24 / 55
Ch. 49,
Study online at https://quizlet.com/_86stsg
retinal vessels
d. ulceration and amputation of the lower b. coronary artery disease
extremities d. ulceration and amputation of the lower
e. capillary and arteriole membrane extremities
thickening specific to diabetes
The pt with diabetes has been diag-
nosed with autnomic neurupathy. What
problems should the nurse expect to find
in the patient (select all that apply)
b. erectile dysfunction
a. painless foot ulcers
e. vomiting undigested food
b. erectile dysfunction
f. painless myocardial infarction
c. burning foot pain at night
d. loss of fine motor control
e. vomiting undigested food
f. painless myocardial infarction
following the teaching of foot care to a di-
abetic patient, the nurse determines that
additional instruction is needed when
the patients make which statement?
a. I should wash my feet daily with soap
and warm water d. ill know if i have sores or lesions on my
b. i should always wear shoes to protect feet because it will be painful
my feet from injury
c. if my feet are cold, i should wear socks
instead of using a heating pad
d. ill know if i have sores or lesions on my
feet because it will be painful
A 72 year old woman is diagnosed with
diabetes. What does the nurse recognize
about the management of diabetes in
the older adult?
a. it is more difficult to achieve strict glu-
a. it is more difficult to achieve strict glu-
cose control than in younger patients
cose control than in younger patients
b. it usually is not treated unless the pa-
tient becomes severely hyperglycemic
c. it does not include treatment with in-
sulin because of limited dexterity and
vision
25 / 55
Ch. 49,
Study online at https://quizlet.com/_86stsg
d. it usually requires that a younger fami-
ly member be responsible for care of the
patient
Polydipsia and polyuria related to dia-
betes mellitus are primarily due to
a.the release of ketones from cells dur-
ing fat metabolism.
b.fluid shifts resulting from the osmotic
b.fluid shifts resulting from the osmotic
effect of hyperglycemia.
effect of hyperglycemia.
c.damage to the kidneys from exposure
to high levels of glucose.
d.changes in RBCs resulting from at-
tachment of excessive glucose to hemo-
globin.
Which statement would be correct for a
patient with type 2 diabetes who was ad-
mitted to the hospital with pneumonia?
a.The patient must receive insulin thera-
py to prevent ketoacidosis.
b.The patient has islet cell antibodies
d.The patient may have sufficient en-
that have destroyed the pancreas's abil-
dogenous insulin to prevent ketosis but
ity to produce insulin.
is at risk for hyperosmolar hyperglycemic
c.The patient has minimal or absent en-
syndrome.
dogenous insulin secretion and requires
daily insulin injections.
d.The patient may have sufficient en-
dogenous insulin to prevent ketosis but
is at risk for hyperosmolar hyperglycemic
syndrome.
Which are appropriate therapies for pa-
tients with diabetes mellitus (select all
that apply)? a.Use of statins to treat dyslipidemia
a.Use of statins to treat dyslipidemia c.Use of ACE inhibitors to treat
b.Use of diuretics to treat nephropathy nephropathy
c.Use of ACE inhibitors to treat e.Use of laser photocoagulation to treat
nephropathy retinopathy
d.Use of serotonin agonists to decrease
appetite
26 / 55
Ch. 49,
Study online at https://quizlet.com/_86stsg
e.Use of laser photocoagulation to treat
retinopathy
an 18 year old female client, 5'4" tall,
weighting 113 kg, comes to the clinic for
a nonhealing wound on her lower leg,
which she has had for two weeks. Which
disease process should the nurse sus-
b. Type 2 diabetes
pect the client has developed?
a. Type 1 diabetes
b. Type 2 diabetes
c. gestational diabetes
d. acanthosis nigricans
The client diagnosed with Type 1 di-
abetes has a glycosylated hemoglo-
bin (A1C) of 8.1%. which interpretation
should the nurse make based on this
result? c. this result is above recommended lev-
a. this result is below normal levels els
b. this result is within acceptable levels
c. this result is above recommended lev-
els
d. this result is dangerously high
The nurses adminstered 28 units of Hu-
mulin N, an intermediate-acting insulin,
to a client diagnosed with type 1 dia-
betes at 1600. Which intervention should
the nurse implement?
a. ensure the client eats the bedtime a. ensure the client eats the bedtime
snack snack
b. determine how much food the client
ate at lunch
c. perform a glucometer reading at 0700
d. offer the client protein after adminis-
tering insulin
the nurse is discussing the importance
of exercising with a client diagnosed with
type 2 diabetes who diabetes is well
controlled with diet and exercise. Which
27 / 55
Ch. 49,
Study online at https://quizlet.com/_86stsg
information should the nurse include in
the teaching about diabetes?
a. eat a simple carb snack before exer-
cising
b. carry peanut butter crackers when ex- d. perform warm up and cool down exer-
ercising cises
c. encourage the client to walk 20 min-
utes three times a week
d. perform warm up and cool down exer-
cises
The nurse is assessing the feet of a
client with long term type 2 diabetes.
Which assessment data warrant imme-
diate intervention by the nurse?
c. the client has a necrotic big toe
a. the client has crumbling toe nails
b. the client has athletes foot
c. the client has a necrotic big toe
d.the client has thickened toenails
the home health nurse is completing the
admission assessment for a 76 year old
client diagnosed with type 2 diabetes
controlled with 70/30 insulin. Which in-
tervention should be included in the plan
of care
a. assess the clients ability to read small
a. assess the clients ability to read small
print
print
b. monitor the clients serum pt level
c. teach the client how to perform a he-
moglobin A1C test daily
d. instruct the client to check the feet
weekly
the client with type 2 diabetes controlled
with biguanide oral diabetic medication
is scheduled for a CT scan with contrast
of the abdomen to evaluate pancreatic
function. Which intervention should the
nurse implement?
a. provide a high-fat diet 24 hour prior to
28 / 55
Ch. 49,
Study online at https://quizlet.com/_86stsg
test
b. hold the biguanide med for 48 hours
prior to test
b. hold the biguanide med for 48 hours
c. obtain an informed consent form for
prior to test
the test
d. administer pancreatic enzymes prior
to the test
the diabetic educator is teach a
class on diabetes type 1 and is dis-
cussing sick-day rules. Which interven-
tion should the diabetes educator in-
clude in the discussion? select all that
apply a. take the diabetic medication even if
a. take the diabetic medication even if unable to eat the clients normal diabetic
unable to eat the clients normal diabetic diet
diet b. if unable to eat, drink liquids equal to
b. if unable to eat, drink liquids equal to the clients normal caloric diet
the clients normal caloric diet e. call the HCP if glucose levels are high-
c. if is not necessary to notify the HCP if er than 180 mg/dL
ketones are in the urine
d. test blood glucose levels and test urine
ketones once a day and keep a record
e. call the HCP if glucose levels are high-
er than 180 mg/dL
The client received 10 units of Humulin
R, a fast actign insulin, at 0700. At 1030
the UAP tells the nurse the client has
a headache and is really acting "funny."
Which intervention should the nurse im-
plement first?
a. Instruct the UAP to obtain the blood c. go to the clients room and assess the
glucose level client for hypoglycemia
b. Have the client drink 8 oz of orange
juice
c. go to the clients room and assess the
client for hypoglycemia
d. prepare to administer 1 ampule 50%
dextrose IV

29 / 55
Ch. 49,
Study online at https://quizlet.com/_86stsg
The nurse at a freestanding health care
clinic is caring for a 56 year old male
client who is homeless and is a type
2 diabetic controlled with insulin. Which
action is an example of client advocacy?
a. ask the client if he has somewhere he
b. arrange for someone to give him in-
can go and live
sulin at a local homeless shelter
b. arrange for someone to give him in-
sulin at a local homeless shelter
c. notify adult protective services about
the clients situation
d. ask the HCP to take the client off
insulin because he is homeless
The nurse is developing a care plan for
the client diagnosed with type 1 dia-
betes. The nurse identifies the problem
"high risk for hyperglycemia related to
noncompliance with the medication reg-
imen." What statement is an appropriate
short-term goal for the client
a. the client will have a blood glucose
a. the client will have a blood glucose
level between 90 and 140 mg/dL
level between 90 and 140 mg/dL
b. the client will demonstrate appropriate
insulin injection technique
c. the nurse will monitor the clients blood
glucose levels 4 time a day
d. the client will maintain normal kidney
function with 30 ml/hr urine output
The client diagnosed with type 2 dia-
betes is admitted to the intensive care
unit with hyperosmolar hyperglycemia
nonketonic syndrome (HHNS) coma.
Which assessment data should the
c. dry mucous membranes
nurse expect the client to exhibit?
a. Kussmauls respiration
b. kiarrhea and epigastric pain
c. dry mucous membranes
d. ketones breath odor

30 / 55
Ch. 49,
Study online at https://quizlet.com/_86stsg
The elderly client is admitted to the in-
tensive care department diagnosed with
severe HHNS. which collaborative inter-
vention should the nurse include in the
plan of care?
a. infuse 0.9% normal saline IV
a. infuse 0.9% normal saline IV
b. administer intermediate-acting insulin
c. perform a blood glucometer cheeks
daily
d. monitor arterial blood gas results
Which electrolyte replacement should
the nurse anticipate being ordered by the
HCP in the client diagnosed with DKA
who has been admitted to the ICU?
b. potassium
a. glucose
b. potassium
c. calcium
d. sodium
The client diagnosed with HHNS was
admitted yesterday with a blood glucose
level of 780 mg/dL. The clients blood glu-
cose level is now 300 mg/dL. Which in-
tervention should the nurse implement?
d. notify the HCP to obtain an order to
a. increase the regular insulin IV drip
decrease insulin
b. check the patients urine for ketones
c. provide the client with a therapeutic
diabetic meal
d. notify the HCP to obtain an order to
decrease insulin
The client diagnosed with type 1 dia-
betes is found lying unconscious on the
a. administer 50% dextrose IVP
floor of the bathroom. Which intervention
dextrose is only given if the client is found
should the nurse implement first?
unconscious and the nurse suspect hy-
a. administer 50% dextrose IVP
poglycemia. This will arouse the patient
b. notify the HCP
immediately
c. move the client to the ICU
d. check the serum glucose level

31 / 55
Ch. 49,
Study online at https://quizlet.com/_86stsg
Which assessment data indicate the
client diagnosed with DKA is responding
to the medical treatment?
a. the client has tended skin turgor and
dry mucous membranes
b. the client is alert and oriented times
b. the client is alert and oriented times
three
three
c. the clients ABG results are pH 7.29,
PaCO2 44, HCO3 15
d. the clients serum potassium level is
3.3 mEq/L
the nurse is discussing ways to prevent
DKA with the client diagnosed with type
1 diabetes. Which instruction is the most
import to discuss with the client?
a. refer the client to the American Dia-
c. take the prescribed insulin even when
betes Association
unable to eat because of illness
b. do not take an OTC meds
c. take the prescribed insulin even when
unable to eat because of illness
d. explain the need to get the annual flu
and pneumonia vaccines
The charge nurse is making client as-
signments in the ICU. Which client
should be assigned to the most experi-
enced nurse?
a. the client with type 2 diabetes who has
a blood glucose level of 348 mg/dL c. the client with DKA who has multifocal
b. the client diagnosed with type 1 dia- premature ventricular contractions
betes who is experiencing hypoglycemia
c. the client with DKA who has multifocal
premature ventricular contractions
d. the client with HHNS who has a plas-
ma osmolarity of 290 mOsm/L
which arterial blood gas results should.d
the nurse expect in the client diagnosed
with DKA
a. pH 7.34, PaO2 99, PaCO2 48, HCO3
32 / 55
Ch. 49,
Study online at https://quizlet.com/_86stsg
24
b. pH 7.38, PaO2 95, PaCO2 40, HCO3
22
d. pH 7.30, PaO2 90, PaCO2 30, HCO3
c. pH 7.46, PaO2 85, PaCO2 30, HCO3
18
18
d. pH 7.30, PaO2 90, PaCO2 30, HCO3
18
the client is admitted to the ICU di-
agnosed with DKA. Which intervention
should the nurse implement? select all a. maintain adequate ventilation
that apply b. assess fluid volume status
a. maintain adequate ventilation c. administer IV potassium
b. assess fluid volume status d. check for urinary ketones
c. administer IV potassium e. monitor intake and output
d. check for urinary ketones
e. monitor intake and output
When comparing the pathophysiology of
type 1 and type 2 diabetes, which state-
ment would be correct for a patient with
type 2 diabetes who was admitted to the
hospital with pneumonia?
A. the patient must receive insulin ther-
apy to prevent the development of ke-
toacidosis
B. the patient has islet cell antibodies
D
that have destroyed the ability of the pan-
creas to produce insulin
C. The patient has minimal or absent en-
dogenous insulin secretion and requires
daily insulin injections
D. the patient may have sufficient en-
dogenous insulin to prevent ketosis but
is at risk for development of hyperosmo-
lar hyperglycemic syndrome
A 54-year-old patient admitted with type
2 diabetes asks the nurse what "type
2" means. What is the most appropriate
response by the nurse?
33 / 55
Ch. 49,
Study online at https://quizlet.com/_86stsg

A) "With type 2 diabetes, the body of the B) "With type 2 diabetes, insulin secre-
pancreas becomes inflamed." tion is decreased, and insulin resistance
is increased."
B) "With type 2 diabetes, insulin secre-
tion is decreased, and insulin resistance In type 2 diabetes mellitus, the secretion
is increased." of insulin by the pancreas is reduced,
and/or the cells of the body become re-
C) "With type 2 diabetes, the patient is sistant to insulin. The pancreas becomes
totally dependent on an outside source inflamed with pancreatitis. The patient is
of insulin." totally dependent on exogenous insulin
and may have had autoantibodies de-
D) "With type 2 diabetes, the body pro- stroy the ²-cells in the pancreas with type
duces autoantibodies that destroy ²-cells 1 diabetes mellitus.
in the pancreas."
D) Glycosylated hemoglobin level

A glycosylated hemoglobin level detects


the amount of glucose that is bound to
red blood cells (RBCs). When circulating
The nurse caring for a patient hospital-
glucose levels are high, glucose attach-
ized with diabetes mellitus would look
es to the RBCs and remains there for
for which laboratory test result to obtain
the life of the blood cell, which is approx-
information on the patient's past glucose
imately 120 days. Thus the test can give
control?
an indication of glycemic control over ap-
proximately 2 to 3 months.
A) Prealbumin level
B) Urine ketone level
The prealbumin level is used to estab-
C) Fasting glucose level
lish nutritional status and is unrelated to
D) Glycosylated hemoglobin level
past glucose control. The urine ketone
level will only show that hyperglycemia
or starvation is probably currently occur-
ring. The fasting glucose level only indi-
cates current glucose control.
The nurse has been teaching a pa-
tient with diabetes mellitus how to per-
form self-monitoring of blood glucose
(SMBG). During evaluation of the pa-

34 / 55
Ch. 49,
Study online at https://quizlet.com/_86stsg
tient's technique, the nurse identifies a
need for additional teaching when the
patient does what?
A) Chooses a puncture site in the center
of the finger pad.
A) Chooses a puncture site in the center
of the finger pad.
The patient should select a site on the
sides of the fingertips, not on the cen-
B) Washes hands with soap and water to
ter of the finger pad as this area con-
cleanse the site to be used.
tains many nerve endings and would be
unnecessarily painful. Washing hands,
C) Warms the finger before puncturing
warming the finger, and knowing the re-
the finger to obtain a drop of blood.
sults that indicate good control all show
understanding of the teaching.
D) Tells the nurse that the result of 110
mg/dL indicates good control of dia-
betes.
The nurse is evaluating a 45-year-old A) Excessive thirst
patient diagnosed with type 2 diabetes
mellitus. Which symptom reported by the The classic symptoms of diabetes are
patient is considered one of the classic polydipsia (excessive thirst), polyuria,
clinical manifestations of diabetes? (excessive urine output), and polyphagia
(increased hunger).
A) Excessive thirst
B) Gradual weight gain Weight gain, fatigue, and blurred vision
C) Overwhelming fatigue may all occur with type 2 diabetes, but
D) Recurrent blurred vision are NOT classic manifestations.
A 51-year-old patient with diabetes mel-
litus is scheduled for a fasting blood glu-
B) Midnight before the test
cose level at 8:00 AM. The nurse in-
structs the patient to only drink water
Typically, a patient is ordered to be NPO
after what time?
for 8 hours before a fasting blood glucose
level. For this reason, the patient who
A) 6:00 PM on the evening before the
has a lab draw at 8:00 AM should not
test
have any food or beverages containing
B) Midnight before the test
any calories after midnight.
C) 4:00 AM on the day of the test
D)7:00 AM on the day of the test

35 / 55
Ch. 49,
Study online at https://quizlet.com/_86stsg
A patient, who is admitted with diabetes
mellitus, has a glucose level of 380
mg/dL and a moderate level of ketones C) Kussmaul respirations
in the urine. As the nurse assesses for
signs of ketoacidosis, which respiratory In diabetic ketoacidosis, the lungs try to
pattern would the nurse expect to find? compensate for the acidosis by blowing
off volatile acids and carbon dioxide. This
A) Central apnea leads to a pattern of Kussmaul respira-
B) Hypoventilation tions, which are deep and nonlabored.
C) Kussmaul respirations
D) Cheyne-Stokes respirations
The nurse is assisting a patient with new-
ly diagnosed type 2 diabetes to learn di-
etary planning as part of the initial man-
A) Cheese
agement of diabetes. The nurse would
encourage the patient to limit intake of
Cheese is a product derived from animal
which foods to help reduce the percent
sources and is higher in fat and calories
of fat in the diet?
than vegetables, fruit, and poultry. Ex-
cess fat in the diet is limited to help avoid
A) Cheese
macrovascular changes.
B) Broccoli
C) Chicken
D) Oranges
Laboratory results have been obtained
for a 50-year-old patient with a 15-year
A) Increased triglyceride levels
history of type 2 diabetes. Which result
reflects the expected pattern accompa-
Macrovascular complications of dia-
nying macrovascular disease as a com-
betes include changes to large- and
plication of diabetes?
medium-sized blood vessels. They in-
clude cerebrovascular, cardiovascular,
A) Increased triglyceride levels
and peripheral vascular disease. In-
creased triglyceride levels are associat-
B) Increased high-density lipoproteins
ed with these macrovascular changes.
(HDL)
Increased HDL, decreased LDL, and de-
creased VLDL are positive in relation to
C) Decreased low-density lipoproteins
atherosclerosis development.
(LDL)

36 / 55
Ch. 49,
Study online at https://quizlet.com/_86stsg
D) Decreased very-low-density lipopro-
teins (VLDL)
The nurse has taught a patient ad-
mitted with diabetes, cellulitis, and os-
teomyelitis about the principles of foot
care. The nurse evaluates that the pa-
tient understands the principles of foot
care if the patient makes what state- B) "I should look at the condition of my
ment? feet every day."

A) "I should only walk barefoot in nice dry Patients with diabetes mellitus need to
weather." inspect their feet daily for broken areas
that are at risk for infection and delayed
B) "I should look at the condition of my wound healing. Properly fitted (not tight)
feet every day." shoes should be worn at all times. Water
temperature should be tested with the
C) "I am lucky my shoes fit so nice and hands first.
tight because they give me firm support."

D) "When I am allowed up out of bed, I


should check the shower water with my
toes."
A patient is admitted with diabetes mel-
A) The level may be increased as a result
litus, malnutrition, and cellulitis. The pa-
of dehydration that accompanies hyper-
tient's potassium level is 5.6 mEq/L. The
glycemia.
nurse understands that what could be
contributing factors for this laboratory re-
C) The level is consistent with renal in-
sult (select all that apply)?
sufficiency that can develop with renal
nephropathy.
A) The level may be increased as a result
of dehydration that accompanies hyper-
E) This level demonstrates adequate
glycemia.
treatment of the cellulitis and effective
serum glucose control.
B) The patient may be excreting ex-
tra sodium and retaining potassium be-
The additional stress of cellulitis may
cause of malnutrition.
lead to an increase in the patient's serum
glucose levels. Dehydration may cause
C) The level is consistent with renal in-

37 / 55
Ch. 49,
Study online at https://quizlet.com/_86stsg
sufficiency that can develop with renal
nephropathy. hemoconcentration, resulting in elevat-
ed serum readings. Kidneys may have
D) The level may be raised as a result of difficulty excreting potassium if renal in-
metabolic ketoacidosis caused by hyper- sufficiency exists. Finally, the nurse must
glycemia. Correct consider the potential for metabolic ke-
toacidosis since potassium will leave the
E) This level demonstrates adequate cell when hydrogen enters in an attempt
treatment of the cellulitis and effective to compensate for a low pH.
serum glucose control.
C) 10:30 PM to 1:30 AM

The patient received regular insulin 10 Regular insulin exerts peak action in 2 to
units subcutaneously at 8:30 PM for a 5 hours, making the patient most at risk
blood glucose level of 253 mg/dL. The for hypoglycemia between 10:30 PM and
nurse plans to monitor this patient for 1:30 AM.
signs of hypoglycemia at which time re-
lated to the insulin's peak action? Rapid-acting insulin's onset is between
10-30 minutes with peak action and hy-
A) 8:40 PM to 9:00 PM poglycemia most likely to occur between
B) 9:00 PM to 11:30 PM 9:00 PM and 11:30 PM.
C) 10:30 PM to 1:30 AM
D) 12:30 AM to 8:30 AM With intermediate acting insulin, hypo-
glycemia may occur from 12:30 AM to
8:30 AM.
C) Eat 15 g of simple carbohydrates.
A college student is newly diagnosed
with type 1 diabetes. She now has a When the patient with type 1 diabetes is
headache, changes in her vision, and is unsure about the meaning of the symp-
anxious, but does not have her portable toms she is experiencing, she should
blood glucose monitor with her. Which treat herself for hypoglycemia to prevent
action should the campus nurse advise seizures and coma from occurring. She
her to take? should also be advised to check her
blood glucose as soon as possible.
A) Eat a piece of pizza.
B) Drink some diet pop. The fat in the pizza and the diet pop
C) Eat 15 g of simple carbohydrates. would not allow the blood glucose to in-
crease to eliminate the symptoms. The

38 / 55
Ch. 49,
Study online at https://quizlet.com/_86stsg
D) Take an extra dose of rapid-acting extra dose of rapid-acting insulin would
insulin. further decrease her blood glucose.
A 65-year-old patient with type 2 di- C) Cardiac monitoring to detect potassi-
abetes has a urinary tract infection um changes
(UTI). The unlicensed assistive person-
nel (UAP) reported to the nurse that This patient has manifestations of hy-
the patient's blood glucose is 642 mg/dL perosmolar hyperglycemic syndrome
and the patient is hard to arouse. When (HHS). Cardiac monitoring will be need-
the nurse assesses the urine, there are ed because of the changes in the potas-
no ketones present. What collaborative sium level related to fluid and insulin
care should the nurse expect for this pa- therapy and the osmotic diuresis from
tient? the elevated serum glucose level. Rou-
tine insulin would not be enough, and
A) Routine insulin therapy and exercise exercise could be dangerous for this pa-
tient. Extra insulin will be needed. The
B) Administer a different antibiotic for the type of antibiotic will not affect HHS.
UTI. There will be a large amount of IV
fluid administered, but it will be giv-
C) Cardiac monitoring to detect potassi- en slowly because this patient is older
um changes and may have cardiac or renal compro-
mise requiring hemodynamic monitoring
D) Administer IV fluids rapidly to correct to avoid fluid overload during fluid re-
dehydration. placement.
The newly diagnosed patient with type 2 C) Reduces glucose production by the
diabetes has been prescribed metformin liver and enhances insulin sensitivity.
(Glucophage). What should the nurse
tell the patient to best explain how this Metformin is a biguanide that reduces
medication works? glucose production by the liver and en-
hances the tissue's insulin sensitivity.
A) Increases insulin production from the
pancreas. Sulfonylureas and meglitinides increase
insulin production from the pancreas.
B) Slows the absorption of carbohydrate
in the small intestine. ±-
glucosidase inhibitors slow the absorp-
tion of carbohydrate in the intestine.
C) Reduces glucose production by the
liver and enhances insulin sensitivity. Glucagon-like peptide receptor agonists
increase insulin synthesis and release

39 / 55
Ch. 49,
Study online at https://quizlet.com/_86stsg
D) Increases insulin release from the
from the pancreas, inhibit glucagon se-
pancreas, inhibits glucagon secretion,
cretion, and decrease gastric emptying.
and decreases gastric emptying.
The nurse is teaching a patient with type
2 diabetes mellitus about exercise to
help control his blood glucose. The nurse
knows the patient understands when the
patient elicits which exercise plan?
D)"I will take a brisk 30-minute walk 5
A) "I want to go fishing for 30 minutes days per week and do resistance training
each day; I will drink fluids and wear 3 times a week."
sunscreen."
The best exercise plan for the person
B) "I will go running each day when my with type 2 diabetes is for 30 minutes of
blood sugar is too high to bring it back to moderate activity 5 days per week and
normal." resistance training 3 times a week. Brisk
walking is moderate activity. Fishing and
C) "I will plan to keep my job as a teacher teaching are light activity, and running is
because I get a lot of exercise every considered vigorous activity.
school day."

D)"I will take a brisk 30-minute walk 5


days per week and do resistance training
3 times a week."
A patient with type 2 diabetes is sched-
uled for an outpatient coronary arteri- B
ogram. Which information obtained by Rationale: To avoid lactic acidosis, met-
the nurse when admitting the patient in- formin should not be used for 48 hours
dicates a need for a change in the pa- after IV contrast media are administered.
tient's regimen? The other patient data indicate that the
a. The patient's most recent hemoglobin patient is managing the diabetes appro-
A1C was 6%. priately.
b. The patient takes metformin (Glu-
cophage) every morning. Cognitive Level: Application Text Refer-
c. The patient uses captopril (Capoten) ence: p. 1266
for hypertension. Nursing Process: Assessment NCLEX:
d. The patient's admission blood glucose Physiological Integrity
is 128 mg/dl.

40 / 55
Ch. 49,
Study online at https://quizlet.com/_86stsg
The nurse is caring for a patient after a
parathyroidectomy. The nurse would pre- Facial muscle spasms and laryn-
pare to administer IV calcium gluconate gospasms (manifestations of hypocal-
if the patient exhibits which clinical man- cemia)
ifestations?
A patient with newly diagnosed type 2
diabetes mellitus asks the nurse what
"type 2" means in relation to diabetes.
B
The nurse explains to the patient that
Rationale: In type 2 diabetes, the pan-
type 2 diabetes differs from type 1 dia-
creas produces insulin, but the insulin is
betes primarily in that with type 2 dia-
insufficient for the body's needs or the
betes
cells do not respond to the insulin appro-
a. the patient is totally dependent on an
priately. The other information describes
outside source of insulin.
the physiology of type 1 diabetes.
b. there is decreased insulin secretion
and cellular resistance to insulin that is
Cognitive Level: Application Text Refer-
produced.
ence: p. 1255
c. the immune system destroys the pan-
Nursing Process: Implementation
creatic insulin-producing cells.
NCLEX: Physiological Integrity
d. the insulin precursor that is secreted
by the pancreas is not activated by the
liver.
D
Rationale: The patient's impaired fast-
ing glucose indicates prediabetes and
the patient should be counseled about
A patient screened for diabetes at a clin-
lifestyle changes to prevent the develop-
ic has a fasting plasma glucose level of
ment of type 2 diabetes. The patient with
120 mg/dl (6.7 mmol/L). The nurse will
prediabetes does not require insulin or
plan to teach the patient about
the oral hypoglycemics for glucose con-
a. use of low doses of regular insulin.
trol and does not need to self-monitor
b. self-monitoring of blood glucose.
blood glucose.
c. oral hypoglycemic medications.
d. maintenance of a healthy weight.
Cognitive Level: Application Text Refer-
ence: p. 1255
Nursing Process: Planning NCLEX:
Physiological Integrity

41 / 55
Ch. 49,
Study online at https://quizlet.com/_86stsg
During a diabetes screening program, a
patient tells the nurse, "My mother died B
of complications of type 2 diabetes. Can Rationale: Offspring of people with type
I inherit diabetes?" The nurse explains 2 diabetes are at higher risk for devel-
that oping type 2 diabetes. The risk can be
decreased, but not prevented, by main-
a. as long as the patient maintains nor- tenance of normal weight and exercising.
mal weight and exercises, type 2 dia- The risk for children of a person with type
betes can be prevented. 1 diabetes to develop diabetes is higher
b. the patient is at a higher than normal when it is the father who has the disease.
risk for type 2 diabetes and should have Offspring of people with type 2 diabetes
periodic blood glucose level testing. are more likely to develop diabetes than
c. there is a greater risk for children de- offspring of those with type 1 diabetes.
veloping type 2 diabetes when the father
has type 2 diabetes. Cognitive Level: Application Text Refer-
d. although there is a tendency for chil- ence: p. 1256
dren of people with type 2 diabetes to Nursing Process: Implementation
develop diabetes, the risk is higher for NCLEX: Physiological Integrity
those with type 1 diabetes.
A program of weight loss and exer-
cise is recommended for a patient with
D
impaired fasting glucose (IFG). When
Rationale: The patient with IFG is at
teaching the patient about the reason for
risk for developing type 2 diabetes, but
these lifestyle changes, the nurse will tell
this risk can be decreased with lifestyle
the patient that
changes. Glycosylated hemoglobin lev-
a. the high insulin levels associated
els will not be elevated in IFG and the
with this syndrome damage the lining of
Hb A1C test is not included in predia-
blood vessels, leading to vascular dis-
betes testing. Elevated insulin levels do
ease.
not cause the damage to blood vessels
b. although the fasting plasma glucose
that can occur with IFG. The liver does
levels do not currently indicate diabetes,
not produce increased levels of glucose
the glycosylated hemoglobin will be ele-
in IFG.
vated.
c. the liver is producing excessive glu-
Cognitive Level: Application Text Refer-
cose, which will eventually exhaust the
ence: p. 1255
ability of the pancreas to produce insulin,
Nursing Process: Implementation
and exercise will normalize glucose pro-
NCLEX: Physiological Integrity
duction.
d. the onset of diabetes and the associ-
42 / 55
Ch. 49,
Study online at https://quizlet.com/_86stsg
ated cardiovascular risks can be delayed
or prevented by weight loss and exer-
cise.
A
Rationale: Weight loss occurs because
the body is no longer able to absorb glu-
When assessing the patient experienc-
cose and starts to break down protein
ing the onset of symptoms of type 1 dia-
and fat for energy. The patient is thirsty
betes, which question should the nurse
but does not necessarily crave sugar-
ask?
containing fluids. Increased appetite is a
a. "Have you lost any weight lately?"
classic symptom of type 1 diabetes. With
b. "Do you crave fluids containing sug-
the classic symptom of polyuria, urine
ar?"
will be very dilute.
c. "How long have you felt anorexic?"
d. "Is your urine unusually dark-col-
Cognitive Level: Application Text Refer-
ored?"
ence: pp. 1255, 1258
Nursing Process: Assessment NCLEX:
Physiological Integrity
C
Rationale: The glycosylated hemoglobin
(Hb A1C) test shows the overall control
of glucose over 90 to 120 days. A fasting
During a clinic visit 3 months following blood level indicates only the glucose
a diagnosis of type 2 diabetes, the pa- level at one time. Urine glucose testing
tient reports following a reduced-calorie is not an accurate reflection of blood glu-
diet. The patient has not lost any weight cose level and does not reflect the glu-
and did not bring the glucose-monitoring cose over a prolonged time. Oral glucose
record. The nurse will plan to obtain a(n) tolerance testing is done to diagnose di-
a. fasting blood glucose level. abetes, but is not used for monitoring
b. urine dipstick for glucose. glucose control once diabetes has been
c. glycosylated hemoglobin level. diagnosed.
d. oral glucose tolerance test.
Cognitive Level: Application Text Refer-
ence: pp. 1258-1259
Nursing Process: Planning NCLEX:
Physiological Integrity
A patient who has just been diagnosed
with type 2 diabetes is 5 ft 4 in (160 cm)
43 / 55
Ch. 49,
Study online at https://quizlet.com/_86stsg
tall and weighs 182 pounds (82 kg). A
C
nursing diagnosis of imbalanced nutri-
Rationale: The complications of diabetes
tion: more than body requirements is de-
are related to elevated blood glucose,
veloped. Which patient outcome is most
and the most important patient outcome
important for this patient?
is the reduction of glucose to near-nor-
a. The patient will have a diet and exer-
mal levels. The other outcomes are also
cise plan that results in weight loss.
appropriate but are not as high in priority.
b. The patient will state the reasons for
eliminating simple sugars in the diet.
Cognitive Level: Application Text Refer-
c. The patient will have a glycosylated
ence: p. 1273
hemoglobin level of less than 7%.
Nursing Process: Planning NCLEX:
d. The patient will choose a diet that dis-
Physiological Integrity
tributes calories throughout the day.
D
Rationale: The change in exercise will
A college student who has type 1 dia-
affect blood glucose, and the patient will
betes normally walks each evening as
need to monitor glucose carefully to de-
part of an exercise regimen. The stu-
termine the need for changes in diet
dent now plans to take a swimming class
and insulin administration. Because ex-
every day at 1:00 PM. The clinic nurse
ercise tends to decrease blood glucose,
teaches the patient to
patients are advised to eat before exer-
a. delay eating the noon meal until after
cising. Increasing the morning NPH or
the swimming class.
timing the insulin to peak during exer-
b. increase the morning dose of neutral
cise may lead to hypoglycemia, especial-
protamine Hagedorn (NPH) insulin on
ly with the increased exercise.
days of the swimming class.
c. time the morning insulin injection so
Cognitive Level: Application Text Refer-
that the peak occurs while swimming.
ence: p. 1269
d. check glucose level before, during,
Nursing Process: Implementation
and after swimming.
NCLEX: Health Promotion and Mainte-
nance
A patient with type 1 diabetes has re- D
ceived diet instruction as part of the Rationale: Most patients with type 1 di-
treatment plan. The nurse determines a abetes need to plan diet choices very
need for additional instruction when the carefully. Patients who are using inten-
patient says, sified insulin therapy have considerable
a. "I may have an occasional alcoholic flexibility in diet choices but still should
drink if I include it in my meal plan." restrict dietary intake of items such as

44 / 55
Ch. 49,
Study online at https://quizlet.com/_86stsg
fat, protein, and alcohol. The other pa-
b. "I will need a bedtime snack because
tient statements are correct and indicate
I take an evening dose of NPH insulin."
good understanding of the diet instruc-
c. "I will eat meals as scheduled, even
tion.
if I am not hungry, to prevent hypo-
glycemia."
Cognitive Level: Application Text Refer-
d. "I may eat whatever I want, as long as
ence: p. 1268
I use enough insulin to cover the calo-
Nursing Process: Evaluation NCLEX:
ries."
Physiological Integrity
A 1200-calorie diet and exercise are pre-
scribed for a patient with newly diag-
B
nosed type 2 diabetes. The patient tells
Rationale: Exercise is essential to de-
the nurse, "I hate to exercise! Can't I just
crease insulin resistance and improve
follow the diet to keep my glucose under
blood glucose control. Increased ener-
control?" The nurse teaches the patient
gy, improved cardiovascular endurance,
that the major purpose of exercise for
and setting a pattern of success are sec-
diabetics is to
ondary benefits of exercise, but they are
a. increase energy and sense of well-be-
not the major reason.
ing, which will help with body image.
b. facilitate weight loss, which will de-
Cognitive Level: Application Text Refer-
crease peripheral insulin resistance.
ence: p. 1269
c. improve cardiovascular endurance,
Nursing Process: Implementation
which is important for diabetics.
NCLEX: Physiological Integrity
d. set a successful pattern, which will
help in making other needed changes.
The nurse has been teaching the patient
to administer a dose of 10 units of reg- A
ular insulin and 28 units of NPH insulin. Rationale: Rotating sites is no longer
The statement by the patient that indi- necessary because all insulin is now
cates a need for additional instruction is, purified human insulin, and the risk for
a. "I need to rotate injection sites among lipodystrophy is low. The other patient
my arms, legs, and abdomen each day." statements are accurate and indicate
b. "I will buy the 0.5-ml syringes because that no additional instruction is needed.
the line markings will be easier to see."
c. "I should draw up the regular insulin Cognitive Level: Application Text Refer-
first after injecting air into the NPH bot- ence: p. 1262
tle." Nursing Process: Evaluation
d. "I do not need to aspirate the plunger

45 / 55
Ch. 49,
Study online at https://quizlet.com/_86stsg
to check for blood before I inject the in- NCLEX: Health Promotion and Mainte-
sulin." nance
B
Rationale: In the Somogyi effect, the pa-
tient's blood glucose drops in the ear-
ly morning hours (in response to ex-
cess insulin administration), which caus-
A patient with type 1 diabetes has an es the release of hormones that result
unusually high morning glucose mea- in a rebound hyperglycemia. It is impor-
surement, and the health care provider tant to check the blood glucose in the
wants the patient evaluated for possible early morning hours to detect the ini-
Somogyi effect. The nurse will plan to tial hypoglycemia. An increased evening
a. administer an increased dose of NPH NPH dose or holding the nighttime snack
insulin in the evening. will further increase the risk for ear-
b. obtain the patient's blood glucose at ly morning hypoglycemia. Information
3:00 in the morning. about symptoms of hypoglycemia will
c. withhold the nighttime snack and not be as accurate as checking the
check the glucose at 6:00 AM. patient's blood glucose in determining
d. check the patient for symptoms of hy- whether the patient has the Somogyi ef-
poglycemia at 2:00 to 4:00 AM. fect.

Cognitive Level: Application Text Refer-


ence: pp. 1263-1264
Nursing Process: Planning NCLEX:
Physiological Integrity
B
A patient receives a daily injection of Rationale: The greatest insulin effect
70/30 NPH/regular insulin premix at 7:00 with this combination occurs mid after-
AM. The nurse expects that a hypo- noon. The patient is not at a high risk
glycemic reaction is most likely to occur at the other listed times, although hypo-
between glycemia may occur.
a. 8:00 and 10:00 AM.
b. 4:00 and 6:00 PM. Cognitive Level: Comprehension Text
c. 7:00 and 9:00 PM. Reference: p. 1260
d. 10:00 PM and 12:00 AM. Nursing Process: Evaluation NCLEX:
Physiological Integrity
A
Rationale: The hypokalemia associated
46 / 55
Ch. 49,
Study online at https://quizlet.com/_86stsg
Cardiac monitoring is initiated for a pa- with metabolic acidosis can lead to po-
tient in diabetic ketoacidosis (DKA). The tentially fatal dysrhythmias such as ven-
nurse recognizes that this measure is tricular tachycardia and ventricular fib-
important to identify rillation, which would be detected with
a. electrocardiographic (ECG) changes ECG monitoring. Fluid overload, hypov-
and dysrhythmias related to hy- olemia, and cardiovascular collapse are
pokalemia. possible complications of DKA, but car-
b. fluid overload resulting from aggres- diac monitoring would not detect theses.
sive fluid replacement.
c. the presence of hypovolemic shock Cognitive Level: Application Text Refer-
related to osmotic diuresis. ence: p. 1281
d. cardiovascular collapse resulting from Nursing Process: Assessment NCLEX:
the effects of hyperglycemia. Physiological Integrity
C
A diabetic patient is admitted with ke- Rationale: The most urgent patient prob-
toacidosis and the health care provider lem is the hypovolemia associated with
writes all of the following orders. Which DKA, and the priority is to infuse IV fluids.
order should the nurse implement first? The other actions can be accomplished
a. Start an infusion of regular insulin at after the infusion of normal saline is initi-
50 U/hr. ated.
b. Give sodium bicarbonate 50 mEq IV
push. Cognitive Level: Application Text Refer-
c. Infuse 1 liter of normal saline per hour. ence: p. 1280
d. Administer regular IV insulin 30 U. Nursing Process: Implementation
NCLEX: Physiological Integrity
B
A patient with type 1 diabetes who uses Rationale: Infection and other stressors
glargine (Lantus) and lispro (Humalog) increase blood glucose levels and the
insulin develops a sore throat, cough, patient will need to test blood glucose
and fever. When the patient calls the clin- frequently, treat elevations appropriate-
ic to report the symptoms and a blood ly with insulin, and call the health care
glucose level of 210 mg/dl, the nurse provider if glucose levels continue to
advises the patient to be elevated. Discontinuing the glargine
a. use only the lispro insulin until the will contribute to hyperglycemia and may
symptoms of infection are resolved. lead to DKA. Decreasing carbohydrate
b. monitor blood glucose every 4 hours or caloric intake is not appropriate as the
and notify the clinic if it continues to rise. patient will need more calories when ill.
Glycosylated hemoglobins are not used

47 / 55
Ch. 49,
Study online at https://quizlet.com/_86stsg
to test for short-term alterations in blood
c. decrease intake of carbohydrates un- glucose.
til glycosylated hemoglobin is less than
7%. Cognitive Level: Application Text Refer-
d. limit intake to non-calorie-containing ence: p. 1272
liquids until the glucose is within the usu- Nursing Process: Implementation
al range. NCLEX: Health Promotion and Mainte-
nance
A
Rationale: The patient's clinical manifes-
tations are consistent with hypoglycemia
and the initial action should be to check
While hospitalized and recovering from
the patient's glucose with a finger stick
an episode of diabetic ketoacidosis, the
or order a stat blood glucose. If the glu-
patient calls the nurse and reports feel-
cose is low, the patient should ingest a
ing anxious, nervous, and sweaty. Based
rapid-acting carbohydrate, such as or-
on the patient's report, the nurse should
ange juice. Glucagon might be given if
a. obtain a glucose reading using a finger
the patient's symptoms become worse
stick.
or if the patient is unconscious. Candy
b. administer 1 mg glucagon subcuta-
bars contain fat, which would slow down
neously.
the absorption of sugar and delay the
c. have the patient eat a candy bar.
response to treatment.
d. have the patient drink 4 ounces of
orange juice.
Cognitive Level: Application Text Refer-
ence: p. 1282
Nursing Process: Implementation
NCLEX: Physiological Integrity
A
Rationale: Rebound hypoglycemia can
Intramuscular glucagon is administered
occur after glucagon administration, but
to an unresponsive patient for treatment
having a meal containing complex car-
of hypoglycemia. Which action should
bohydrates plus protein and fat will help
the nurse take after the patient regains
prevent hypoglycemia. Orange juice and
consciousness?
nonfat milk will elevate blood sugar rapid-
a. Give the patient a snack of cheese and
ly, but the cheese and crackers will sta-
crackers.
bilize blood sugar. Administration of glu-
b. Have the patient drink a glass of or-
cose intravenously might be used in pa-
tients who were unable to take in nu-

48 / 55
Ch. 49,
Study online at https://quizlet.com/_86stsg
trition orally. The patient should be as-
sessed for symptoms of hypoglycemia
ange juice or nonfat milk.
after glucagon administration.
c. Administer a continuous infusion of
5% dextrose for 24 hours.
Cognitive Level: Application Text Refer-
d. Assess the patient for symptoms of
ence: p. 1282
hyperglycemia.
Nursing Process: Implementation
NCLEX: Physiological Integrity
C
Rationale: Hypoglycemic unawareness
A type 1 diabetic patient who was admit- is caused by autonomic neuropathy,
ted with severe hypoglycemia and treat- which would also cause delayed gastric
ed tells the nurse, "I did not have any of emptying. Calcium-channel blockers are
the usual symptoms of hypoglycemia." not associated with hypoglycemic un-
Which question by the nurse will help awareness, although -adrenergic block-
identify a possible reason for the pa- ers can prevent patients from hav-
tient's hypoglycemic unawareness? ing symptoms of hypoglycemia. Skin
a. "Do you use any calcium-channel changes can occur with diabetes, but
blocking drugs for blood pressure?" these are not associated with autonomic
b. "Have you observed any recent skin neuropathy. If the patient can feel painful
changes?" areas on the feet, neuropathy has not
c. "Do you notice any bloating feeling occurred.
after eating?"
d. "Have you noticed any painful new Cognitive Level: Application Text Refer-
ulcerations or sores on your feet?" ence: p. 1281
Nursing Process: Assessment NCLEX:
Physiological Integrity
A patient with type 2 diabetes has sen- B
sory neuropathy of the feet and legs and Rationale: The patient is taught to avoid
peripheral vascular disease evidenced high heels and that leather shoes are
by decreased peripheral pulses and de- preferred. The feet should be washed,
pendent rubor. The nurse teaches the but not soaked, in warm water daily.
patient that Heating pad use should be avoided.
a. the feet should be soaked in warm Commercial callus and corn removers
water on a daily basis. should be avoided; the patient should
b. flat-soled leather shoes are the best see a specialist to treat these problems.
choice to protect the feet from injury.
c. heating pads should always be set at Cognitive Level: Application Text Refer-

49 / 55
Ch. 49,
Study online at https://quizlet.com/_86stsg
a very low temperature. ence: p. 1287
d. over-the-counter (OTC) callus re- Nursing Process: Implementation
mover may be used to remove callus and NCLEX: Health Promotion and Mainte-
prevent pressure. nance
B
A newly diagnosed type 1 diabetic pa-
Rationale: Blood sugar increases after
tient likes to run 3 miles several morn-
meals, so this will be the best time to
ings a week. Which teaching will the
exercise. NPH insulin will not peak until
nurse implement about exercise for this
mid-afternoon and is safe to take before
patient?
a morning run. Running can be done
a. "You should not take the morning NPH
in either the morning or afternoon. If
insulin before you run."
the glucose is very elevated, the patient
b. "Plan to eat breakfast about an hour
should postpone the run.
before your run."
c. "Afternoon running is less likely to
Cognitive Level: Application Text Refer-
cause hypoglycemia."
ence: p. 1269
d. "You may want to run a little farther if
Nursing Process: Implementation
your glucose is very high."
NCLEX: Physiological Integrity
B
Rationale: Oral contraceptive use may
The health care provider orders oral glu-
falsely elevate oral glucose tolerance
cose tolerance testing for a patient seen
test (OGTT) values. A viral 2 months pre-
in the clinic. Which information from the
viously illness may be associated with
patient's health history is most impor-
the onset of type 1 diabetes but will not
tant for the nurse to communicate to the
falsely impact the OGTT. Exercise and a
health care provider?
family history of diabetes both can affect
a. The patient had a viral illness 2
blood glucose but will not lead to mis-
months ago.
leading information from the OGTT.
b. The patient uses oral contraceptives.
c. The patient runs several days a week.
Cognitive Level: Application Text Refer-
d. The patient has a family history of
ence: p. 1267
diabetes.
Nursing Process: Assessment NCLEX:
Physiological Integrity
B
Rationale: The nurse should assess the
Which of these laboratory values noted
patient with a blood glucose level of 52
by the nurse when reviewing the chart of
mg/dl for symptoms of hypoglycemia,
and give the patient some carbohy-
50 / 55
Ch. 49,
Study online at https://quizlet.com/_86stsg
drate-containing beverage such as or-
ange juice. The other values are within
a diabetic patient indicates the need for
an acceptable range for a diabetic pa-
further assessment of the patient?
tient.
a. Fasting blood glucose of 130 mg/dl
b. Noon blood glucose of 52 mg/dl
Cognitive Level: Application Text Refer-
c. Glycosylated hemoglobin of 6.9%
ence: pp. 1281-1282
d. Hemoglobin A1C of 5.8%
Nursing Process: Assessment NCLEX:
Physiological Integrity
After the home health nurse has taught
a patient and family about how to use
glargine and regular insulin safely, which A
action by the patient indicates that the Rationale: Insulin can be stored at
teaching has been successful? room temperature for 4 weeks. Glargine
a. The patient disposes of the open in- should not be mixed with other insulins
sulin vials after 4 weeks. or prefilled and stored. Freezing alters
b. The patient draws up the regular in- the insulin molecule and should not be
sulin in the syringe and then draws up done.
the glargine.
c. The patient stores extra vials of both Cognitive Level: Application Text Refer-
types of insulin in the freezer until need- ence: p. 1261
ed. Nursing Process: Evaluation NCLEX:
d. The patient's family prefills the sy- Physiological Integrity
ringes weekly and stores them in the
refrigerator.
D
Rationale: Patients should be taught not
to administer insulin into a site that will be
The nurse teaches the diabetic patient
exercised because exercise will increase
who rides a bicycle to work every day to
the rate of absorption. The thigh, buttock,
administer morning insulin into the
and arm are all exercised by riding a
a. thigh.
bicycle.
b. buttock.
c. arm.
Cognitive Level: Application Text Refer-
d. abdomen.
ence: p. 1262
Nursing Process: Implementation
NCLEX: Physiological Integrity

51 / 55
Ch. 49,
Study online at https://quizlet.com/_86stsg
Which statement by a nurse to a patient
newly diagnosed with type 2 diabetes is
correct?
ANS: C
For some patients with type 2 diabetes,
a. Insulin is not used to control blood
changes in lifestyle are sufficient to
glucose in patients with type 2 diabetes.
achieve blood glucose control. Insulin is
b. Complications of type 2 diabetes are
frequently used for type 2 diabetes, com-
less serious than those of type 1 dia-
plications are equally severe as for type
betes.
1 diabetes, and type 2 diabetes is usually
c. Changes in diet and exercise may
diagnosed with routine laboratory testing
control blood glucose levels in type 2
or after a patient develops complications
diabetes.
such as frequent yeast infections.
d. Type 2 diabetes is usually diagnosed
when the patient is admitted with a hy-
perglycemic coma.
A 28-year-old male patient with type 1
diabetes reports how he manages his
exercise and glucose control. Which be-
havior indicates that the nurse should
ANS: D
implement additional teaching?
When the patient is ketotic, exercise may
result in an increase in blood glucose
a. The patient always carries hard can-
level. Type 1 diabetic patients should be
dies when engaging in exercise.
taught to avoid exercise when ketosis is
b. The patient goes for a vigorous walk
present. The other statements are cor-
when his glucose is 200 mg/dL.
rect
c. The patient has a peanut butter sand-
wich before going for a bicycle ride.
d. The patient increases daily exercise
when ketones are present in the urine.
The nurse is assessing a 22-year-old
ANS: C
patient experiencing the onset of symp-
Weight loss occurs because the body is
toms of type 1 diabetes. Which question
no longer able to absorb glucose and
is most appropriate for the nurse to ask?
starts to break down protein and fat for
energy. The patient is thirsty but does not
a. "Are you anorexic?"
necessarily crave sugar-containing flu-
b. "Is your urine dark colored?"
ids. Increased appetite is a classic symp-
c. "Have you lost weight lately?"
tom of type 1 diabetes. With the classic
d. "Do you crave sugary drinks?"

52 / 55
Ch. 49,
Study online at https://quizlet.com/_86stsg
symptom of polyuria, urine will be very
dilute
A 55-year-old female patient with type
2 diabetes has a nursing diagnosis of
imbalanced nutrition: more than body re-
quirements. Which goal is most impor-
ANS: A
tant for this patient?
The complications of diabetes are relat-
ed to elevated blood glucose, and the
a. The patient will reach a glycosylated
most important patient outcome is the re-
hemoglobin level of less than 7%.
duction of glucose to near-normal levels.
b. The patient will follow a diet and exer-
The other outcomes also are appropriate
cise plan that results in weight loss.
but are not as high in priority
c. The patient will choose a diet that dis-
tributes calories throughout the day.
d. The patient will state the reasons for
eliminating simple sugars in the diet.
ANS: B
Which action should the nurse take after
Rebound hypoglycemia can occur after
a 36-year-old patient treated with intra-
glucagon administration, but having a
muscular glucagon for hypoglycemia re-
meal containing complex carbohydrates
gains consciousness?
plus protein and fat will help prevent
hypoglycemia. Orange juice and nonfat
a. Assess the patient for symptoms of
milk will elevate blood glucose rapidly,
hyperglycemia.
but the cheese and crackers will stabilize
b. Give the patient a snack of peanut
blood glucose. Administration of IV glu-
butter and crackers.
cose might be used in patients who were
c. Have the patient drink a glass of or-
unable to take in nutrition orally. The pa-
ange juice or nonfat milk.
tient should be assessed for symptoms
d. Administer a continuous infusion of
of hypoglycemia after glucagon adminis-
5% dextrose for 24 hours.
tration
Which question during the assessment
ANS: A
of a diabetic patient will help the nurse
Autonomic neuropathy can cause de-
identify autonomic neuropathy?
layed gastric emptying, which results in a
bloated feeling for the patient. The other
a. "Do you feel bloated after eating?"
questions are also appropriate to ask but
b. "Have you seen any skin changes?"
would not help in identifying autonomic
c. "Do you need to increase your insulin
neuropathy
dosage when you are stressed?"
53 / 55
Ch. 49,
Study online at https://quizlet.com/_86stsg
d. "Have you noticed any painful new
ulcerations or sores on your feet?"
Which information will the nurse include
in teaching a female patient who has pe-
ripheral arterial disease, type 2 diabetes, ANS: A
and sensory neuropathy of the feet and The patient is taught to avoid high heels
legs? and that leather shoes are preferred. The
feet should be washed, but not soaked,
a. Choose flat-soled leather shoes. in warm water daily. Heating pad use
b. Set heating pads on a low tempera- should be avoided. Commercial callus
ture. and corn removers should be avoided.
c. Use callus remover for corns or callus- The patient should see a specialist to
es. treat these problems
d. Soak feet in warm water for an hour
each day.
After change-of-shift report, which pa-
tient should the nurse assess first?
ANS: B
Because the brain requires glucose to
a. 19-year-old with type 1 diabetes who
function, untreated hypoglycemia can
has a hemoglobin A1C of 12%
cause unconsciousness, seizures, and
b. 23-year-old with type 1 diabetes who
death. The nurse will rapidly assess and
has a blood glucose of 40 mg/dL
treat the patient with low blood glucose.
c. 40-year-old who is pregnant and
The other patients also have symptoms
whose oral glucose tolerance test is 202
that require assessments and/or inter-
mg/dL
ventions, but they are not at immediate
d. 50-year-old who uses exenatide (Byet-
risk for life-threatening complications
ta) and is complaining of acute abdomi-
nal pain
ANS: B, C, D, F
To monitor for complications in a patient
Blood pressure, serum creatinine,
with type 2 diabetes, which tests will the
urine testing for microalbuminuria, and
nurse in the diabetic clinic schedule at
monofilament testing of the foot are
least annually (select all that apply)?
recommended at least annually to
screen for possible microvascular and
a. Chest x-ray
macrovascular complications of dia-
b. Blood pressure
betes. Chest x-ray and CBC might be
c. Serum creatinine
ordered if the diabetic patient presents
d. Urine for microalbuminuria
with symptoms of respiratory or infec-
54 / 55
Ch. 49,
Study online at https://quizlet.com/_86stsg
e. Complete blood count (CBC) tious problems but are not routinely in-
f. Monofilament testing of the foot cluded in screening
In which order will the nurse take these
steps to prepare NPH 20 units and reg-
ular insulin 2 units using the same sy- ANS:
ringe? (Put a comma and a space be- A, D, E, B, C
tween each answer choice [A, B, C, D, When mixing regular insulin with NPH,
E]). it is important to avoid contact between
a. Rotate NPH vial. the regular insulin and the additives in
b. Withdraw regular insulin. the NPH that slow the onset, peak, and
c. Withdraw 20 units of NPH. duration of activity in the longer-acting
d. Inject 20 units of air into NPH vial. insulin.
e. Inject 2 units of air into regular insulin
vial

55 / 55

You might also like